Sunteți pe pagina 1din 87

Test 1 Sample Essay - Score of 6

Every person has his or her own idea of ethics and morality, regardless of objective truth. Authority figures, whether teachers, heads of nations, or other positions of power, may not always be in line with that morality. In order to determine whether an authority figure is doing what is just, individuals must speak out when they perceive injustice. Oftentimes, that means questioning authority. Dystopic novels often portray systems of national leadership in which questioning governmental authority is explicitly forbidden. George Orwells novel 1984 is such a story. The government is known collectively as "Big Brother," who is a symbolic entity continually watching over every citizen. The language, Newspeak, is constantly being updated to control peoples expression and thereby control their thoughts. Anyone caught even thinking something other than Big Brother would want them to think are arrested by the Thought Police. Under this regime, there are virtually no independent thinkers. Every piece of information given to the people comes through the government, which has the power to change facts and history with utter impunity. The main character, Winston, manages to catch on, but in the end, he is finally brainwashed by torture. In this society of obedient drones, truth is irrelevant because there are no lies. When a student goes through formal education, he must be on his guard when it comes to what he absorbs. In the midst of objective facts, all too often a teacher may attempt to indoctrinate him with the teachers own ideology. One need only look at the majority of United States universities. There are thousands of easily-accessed articles exposing documented incidences of teachers boldly instilling in their students the liberal ideology. Universities are cesspools of leftist professors who punish students who express differences of opinion. Yet without those few questioning students, the rest of the student body would be subjected to unadulterated liberal lectures, with nary an opposing view in earshot. In order to form proper opinions, students must be exposed to both sides of an issue, and both sides must be questioned so as to ascertain the truth of each. Leaders of other countries must also be questioned by nonresidents so as to discover precisely what is happening under their authority. Score Explanation This essay demonstrates outstanding critical thinking by effectively and insightfully developing a point of view (In order to determine whether an authority figure is doing what is just, individuals must speak out when they perceive injustice. Oftentimes, that means questioning authority ), providing clearly appropriate reasons and examples from literature and current events as support. The essay is well organized and clearly focused on authorities who "indoctrinate" others, displaying clear coherence and smooth progression of ideas (Universities are cesspools of leftist professors who punish students who express differences of opinion. Yet without those few questioning students, the rest of the student body would be subjected to unadulterated liberal lectures, with nary an opposing view in earshot. In order to form proper opinions, students must be exposed to both sides of an issue, and both sides must be questioned so as to ascertain the truth of each). Using a varied, accurate, and apt vocabulary, the response exhibits skillful use of language, and there is meaningful variety in sentence structure throughout (Every piece of information given to the people comes through the government, which has the power to change facts and history with utter impunity ). Demonstrating clear and consistent mastery, this essay receives a score of 6.

Sample Essay - Score of 5


While respect should be given to figures of authority, they should not be followed blindly. People with power need to be kept in check, and if those who follow them dont do this no one will. For this reason it is not the option but rather the duty of people under someone elses power to question them and their authority. When people see something that they feel is wrong, they must speak up. If a student notices that a teacher has made a mistake, they must say something. Yes, they are second guessing the teacher, but it is for the benefit of the whole class. It is better to speak up then to let the class go on and to let all the other students be misinformed. On a larger scale, citizens must speak up when they see their leaders doing something that they do not agree with. They do this at a risk to themselves, though. Questioning authority can make one look ungreatful for that authority, but that is a risk many are willing to take. John Kerry, for example, stood up and spoke out against the war in Vietnam. Many people saw

this as unpatriotic and anti-American, but he did it anyway because he believed that his government, and the men with authority, needed to know that he disagreed with them. Many people in authority make very important decisions. These decisions usually directly affect many people. If someone is being directly affected by a decision then it is their right to ask questions or challenge it. If leaders dont hear from the people, then they will not know what the people want, so communication with and questioning of people in authority is crucial to a working relationship. This is demonstrated in the Senate and the House. People write to their representatives and ask them questions and challenge decisions that they have made. All of these things help these powerful men make good decisions. While all of this challenging and questioning is good, figures of authority must be respected. They have a lot of responsibility and deserve respect for what they do. Also, if a person in authority is not respected by a person or group of people, they will not take comments or complaints from those people seriously. George Bush wont take advice from Sorryeverybody.com because he knows that they make a mockery of his re-election. Respect has to be mutual and people have to be vocal in order for people in authority to be able to do a good job and keep the opinion of the people in mind. Score Explanation This essay effectively develops a point of view (. . . it is not the option but rather the duty of people under someone elses power to question them and their authority ) and demonstrates strong critical thinking, providing appropriate reasons and examples from personal observation, history, and government to support the position. The essay is focused and well organized as it moves through its examples, displaying coherence and progression of ideas ( If a student notices that a teacher has made a mistake, they must say something. Yes, they are second guessing the teacher, but it is for the benefit of the whole class. It is better to speak up then to let the class go on and to let all the other students be misinformed). Through the use of appropriate vocabulary, the response exhibits facility in the use of language, and variety in sentence structure appears throughout ( If leaders dont hear from the people, then they will not know what the people want, so communication with and questioning of people in authority is crucial to a working relationship). To earn a score of 6, the writer should use critical thinking and additional detailed evidence to enrich the examples, which are somewhat general. This essay demonstrates reasonably consistent mastery and merits a score of 5.

Sample Essay - Score of 5


When any group, organization, or people choose a leader, they expect to be governed well. Throughout history powerful nations have been reduced to rubble because of poor leadership. Unfortunately, there was not much the people could do because speaking out against their ruler was illegal. Although most governing bodies have the best interest of the people in mind, the real experts on the issues of the people are the people themselves. In the United States people not only have, but cannot be denied the right of questioning authority. Far from rebellious, this ability to voice concern is important because it appeases the people, rights wrongs, and makes governing more efficient. Any governed people will be easier to mange if they are happy. Happy people are more productive. Therefore, happy people benefit the entire country. In order for people to feel content they have to feel like they are heard. Everyones opinion matters and is important. When people feel that their important matter is being ignored they become disgruntled. In order to affectively manage a people it is very important to make sure they are heard. Their opinion can prove to be invaluable. Another reason that voicing concern to the government is because it can lead to the betterment of the whole. In the 1960s the United States witnessed the Civil Rights Movement. Prior to that point the country was segregated. This was an ugly time when whites enjoyed the better of everything. Black people felt this injustice and did something about it. They spoke out against this evil institution and spurred change. Because of their bravery everyone is on an equal playing field. Governing becomes more efficient when people can speak out against it. A ruling body will not always know when part of the system is not working anymore. When people are allowed to talk about their difficulties with healthcare and public school systems those problems can be rectified. Not all comments about the governing body are going to be positive. It is the negative ones that can sometimes make the difference. A person with a problem and a voice to call it out is the most invaluabl asset to a country. The people know best what will make their lives more productive. It is extremely important that they be allowed to speak out about the ineffective measures in order to develop new ones. This right should be undeniable to all people.

Score Explanation Demonstrating strong critical thinking, this essay effectively develops a point of view on questioning authority (Far from rebellious, this ability to voice concern is important because it appeases the people, rights wrongs, and makes governing more efficient), providing appropriate reasons and examples to support the position. Well organized and focused on its three reasons why "voicing concern to the government" is important, the response exhibits coherence and progression of ideas ( Throughout history powerful nations have been reduced to rubble because of poor leadership. Unfortunately, there was not much the people could do because speaking out against their ruler was illegal. Although most governing bodies have the best interest of the people in mind, the real experts on the issues of the people are the people themselves), despite some repetitive and general ideas. Using appropriate vocabulary, the essay displays facility in the use of language and variety in sentence structure ( Black people felt this injustice and did something about it. . . . Because of their bravery everyone is on an equal playing field). In order to receive a score of 6, the writer needs to develop the one specific example more fully, and should highlight the connections among the examples rather than treating them as three distinct ideas. Demonstrating reasonably consistent mastery, this essay receives a score of 5.

Sample Essay - Score of 4


In order for societies to grow, they must adapt to new ideas and opinions. Although it is difficult to decide what is best for a society, an authoritative figure must make these decisions. However, sometimes it is difficult to make the right decisions. It is, then, important to question ideas and decisions of authority figures because without new ideas, there would never be change in a society. Without change, it is difficult for societies to grow and prosper. It is necessary for one person to start a revolution in order for changes to occurr. For example, during a difficult time of discrimination, Rosa Parks took a stand. She refused to sit in the back of the bus so a white person could have her seat. Although this act may have seemed insignificant, it really started to get more and more African-Americans to speak out against authority. Eventually, many of the laws were rewritten so that freedom is equal for everyone. It was completely necessary for one woman to question the authority, or our society may have never grown out of that frame of mind. Although it may seem difficult to speak out against ways you have lived by your whole life, oftentimes it is better to go against a society and stand up for what you believe in, rather than following wrongful laws and regulations. In The Adventures of Huckelberry Finn, Huck has been taught his entire life that Jim is a slave and should be treated as such. However, through their journies together, Jim and Huck develop a new appreciation for each other. Huck begins to view Jim as less of the animal society has called him and more of a friend. In desperate times, Huck would come up with clever stories and lies so that Jim would not be arrested and returned to slavery. Huck went against everything he knew so he could help a friend. Hucks mind was one of a few that went against a society that believed the color of your skin was more important than any other characteristic. When more and more people come together with a common goal, it is easier to question and change a society. Without questioning authority, it would not be possible to have variety in a society. In The Giver, Jonas world is a community of "sameness" and regularity. There is no pain or love and there is no color or world beyond their tiny community. Score Explanation Demonstrating competent critical thinking, this essay develops a point of view ( It is, then, important to question ideas and decisions of authority figures because without new ideas, there would never be change in a society) and supports the position with adequate examples from history and literature. Generally organized and focused around its examples, the essay exhibits some coherence and progression of ideas (In The Adventures of Huckelberry Finn, Huck has been taught his entire life that Jim is a slave and should be treated as such. However, through their journies together, Jim and Huck develop a new appreciation for each other). The essay displays adequate facility in the use of language, using generally appropriate vocabulary and a varied sentence structure ( Although this act may have seemed insignificant, it really started to get more and more African-Americans to speak out against authority). To earn a higher score, the writer should add specific details: both examples, while adequate, are too vague and general to really be effective. This essay demonstrates adequate mastery and receives a score of 4.

Sample Essay - Score of 4

When a person is placed in a position of authority, he or she is considered to be an expert or a leader in his or her field. Despite this, once in a while, every leader, director, or president might make a bad decision or make a mistake. Being wrong once in a while is part of being human. This is why that sometimes one might find it necessary or important to question the ideas and decisions of people in positions of authority. Respect for an authority figure is obviously needed in order for a business, group, or organization to work properly. Although, there are times when questioning the people in charge is necessary. Even I have been in situations where authority figures, such as a parent or a group leader, havent always had the best idea or made the right decision. One time, the coach of my soccer team had decided that we were going to play in a soccer tournement down in New Jersey. To make sure all of the team members arrived at the same time, he told us that we were going to take a plane down. My teammates and I werent thrilled with the idea because we didnt want to spend the money for a plane ticket. I began to think and came up with the idea of us all taking a bus down to New Jersey. It was cheaper for all of us and we would still all be together. I then mentioned the idea to my coach and he agreed with it, even though I questioned his authority. We ended up renting a bus, paying less money, and truly having more fun. By questioning or giving new ideas to people in authority, we ourselves become better thinkers. Because I gave that idea to my coach, he now asks me how we can travel down to all of our tournements. By giving new ideas or questioning old ones, it forces all concerned to defend the old ideas and decisions or to consider new ones. These new thoughts could possibly even correct past errors or put a stop to wrong actions in the future. Whether one questions a CEO of a major corporation, a daughter gives an idea her mother, or a student corrects his teacher, it is important to question the ideas and decisions of people in positions of authority. Score Explanation This essay develops its point of view (. . . it is important to question the ideas and decisions of people in positions of authority) with competent critical thinking, using an adequate example from personal experience to support the position. The essay is generally organized and focused on its argument, displaying some coherence and progression of ideas ( I began to think and came up with the idea of us all taking a bus down to New Jersey. . . . I then mentioned the idea to my coach and he agreed with it, even though I questioned his authority. We ended up renting a bus, paying less money, and truly having more fun). The essay displays adequate facility in the use of language with generally appropriate vocabulary and some variety in sentence structure ( To make sure all of the team members arrived at the same time, he told us that we were going to take a plane down ). In order to achieve a higher score, the writer needs to use critical thinking and more detail to enhance the example, or provide additional examples, to further develop the point of view. Demonstrating adequate mastery, this response merits a score of 4.

Section #2: View Explanations 1 Explanation for Correct Answer A. Choice (A) is correct. Foresight is the ability to see ahead. According to the sentence, Aleksandr Solzhenitsyn predicted, or stated in advance, that his books would someday appear in Russia. It makes sense to say that Solzhenitsyns foresight was very keen, or sharp, as his prediction turned out to be accurate ("he accurately predicted . . ."). Choice (B) is incorrect. Nostalgia is a wistful and sentimental longing to return to the past. According to the sentence, Aleksandr Solzhenitsyn predicted, or stated in advance, that his books would someday appear in Russia. Whereas nostalgia relates to events that occurred in the past, Solzhenitsyn predicted what would happen in the

future. Therefore, it does not make sense to describe Solzhenitsyns accurate prediction as a demonstration of his keen nostalgia. Choice (C) is incorrect. Folly is the lack of good sense or the inability to see ahead. According to the sentence, Aleksandr Solzhenitsyn predicted, or stated in advance, that his books would someday appear in Russia. Solzhenitsyns accurate prediction indicates that he has a keen, or sharp, ability to see ahead; therefore, it does not make sense to describe his prediction as an indication of his inability to see ahead. Choice (D) is incorrect. Despair is an utter loss of hope. According to the sentence, Aleksandr Solzhenitsyn predicted, or stated in advance, that his books would someday appear in Russia. Although a person could lose hope and still have the ability to make predictions, making predictions does not directly relate to despair. It does not make sense to describe Solzhenitsyns accurate prediction as an indication of his keen, or sharp, despair. Choice (E) is incorrect. Artistry is artistic ability. According to the sentence, Aleksandr Solzhenitsyn predicted, or stated in advance, that his books would someday appear in Russia. Solzhenitsyn has written books, so he may be considered artistic; however, making predictions is not really an artistic ability. It does not make sense to describe Solzhenitsyns accurate prediction as an indication of his keen, or sharp, artistry. 2 Explanation for Correct Answer B. Choice (B) is correct. In this context, candid means characterized by honest sincerity and a lack of deception. The sentence indicates that Dorothea Langes simple and direct photographs reflect a bygone social milieu, or a social environment of the past; if Langes photographs contain simple, straightforward images, it makes sense to say that they provide a candid, or honest and sincere, reflection of a certain place and time. Choice (A) is incorrect. Intricate means complicated or difficult to understand. The sentence indicates that Dorothea Langes simple and direct photographs reflect a bygone social milieu, or a social environment of the past. It is unlikely that simple, direct images would provide an intricate reflection of a certain place and time; images that are simple are by definition not complicated. Choice (C) is incorrect. Ostentatious means characterized by showiness and vanity. The sentence indicates that Dorothea Langes simple and direct photographs reflect a bygone social milieu, or a social environment of the past. It is unlikely that simple, direct images would provide an ostentatious reflection of a certain place and time; images that are simple are by definition not showy.

Choice (D) is incorrect. Fictional means imagined or fabricated. The sentence states that Dorothea Langes simple and direct photographs reflect a bygone social milieu, or a social environment of the past. Nothing in the sentence indicates that the images in the photographs provide a fictional, or fabricated, reflection; on the contrary, the phrase simple and direct suggests that Langes photographs are straightforward, presenting a glimpse of how things really were. Choice (E) is correct. Convoluted means intricate or complicated. The sentence indicates that Dorothea Langes simple and direct photographs reflect a bygone social milieu, or a social environment of the past. It is unlikely that simple, direct images would provide a convoluted reflection of a certain place and time; images that are simple are by definition not intricate or complicated. 3 Explanation for Correct Answer A. Choice (A) is correct. Someone who is capricious is characterized by impulsive behavior and sudden actions. Because Kate is impulsive and acts on sudden whims, it makes sense that her friends would label her capricious. Choice (B) is incorrect. To be bombastic is to be pompous or use fancy speech and writing. The structure of the sentence suggests that Kates friends would describe her in a way that reflects her impulsive nature and sudden whims. Someone who is pompous may also be impulsive and have sudden whims, but there is no direct connection between behaving impulsively and being pompous. Therefore, there is no reason to believe Kates friends would label her bombastic. Choice (C) is incorrect. To be loquacious is to be extremely talkative. The structure of the sentence suggests that Kates friends would describe her in a way that reflects her impulsive nature and sudden whims. Someone who is extremely talkative may also be impulsive and have sudden whims, but there is no direct connection between behaving impulsively and being very talkative. Therefore, there is no reason to believe Kates friends would label her loquacious. Choice (D) is incorrect. Someone who is dispassionate is not influenced by personal feelings or emotions. The structure of the sentence suggests that Kates friends would describe her in a way that reflects her impulsive nature and sudden whims. If Kate is impulsive, she certainly might make sudden decisions based on her feelings, but such behavior would lead her friends to label her passionate, not dispassionate. Choice (E) is incorrect. To be decorous is to be characterized by good taste and socially appropriate behavior. The structure of the sentence suggests that Kates friends would describe her in a way that reflects her impulsive nature and sudden whims. Although it is possible that a person could be decorous and make impulsive decisions, impulsive people are not necessarily decorous; in fact, in order to avoid inappropriate behavior a decorous person would probably make decisions carefully.

There is no reason to believe that Kates behavior would cause her friends to label her decorous. 4 Explanation for Correct Answer D. Choice (D) is correct. In this context, deliberate means characterized by careful thought. Instinctive means spontaneous or unplanned. The phrase rather than indicates that there is a contrast between the term visceral and the term that fits the first blank. The term deliberate supports this contrast, because a visceral decision is one that is made spontaneously, without thinkingit is not carefully thought out. Additionally, the phrase that is indicates that the part of the sentence after the semicolon restates the idea in the first part of the sentence: Canady believed that people choose their careers instinctively, or spontaneously, rather than rationally. Choice (A) is incorrect. In this context, emotional means directly connected with ones feelings. Intellectual means rational or guided by intellect. The phrase rather than indicates that there is a contrast between the term visceral and the term that fits the first blank. The term emotional does not support this contrast, because a visceral decision is one that is made spontaneously, and such a decision might very well be connected to ones emotions. Additionally, the phrase that is indicates that the part of the sentence after the semicolon restates the idea in the first part of the sentence. The statement that Canady believed career choices to be intellectual and not rational is not a restatement of the previous idea. The statement is also illogical. Choice (B) is incorrect. A chance judgment is one that occurs unpredictably and without a known cause. Random means made, done, or chosen without a definite plan or purpose. The phrase rather than indicates that there is a contrast between the term visceral and the term that fits the first blank. The term chance does not support this contrast, because a visceral decision is one that is made spontaneously, and such a decision might be unpredictable. The part of the sentence after the semicolon makes sense, but the part of the sentence before the semicolon features a central flaw: a visceral decision is not the opposite of a chance judgment. Choice (C) is incorrect. Intuitive means known by intuition, or an immediate understanding or insight. Impulsive means arising from an impulse, or a sudden, spontaneous inclination. The phrase rather than indicates that there is a contrasting relationship between the term visceral and the term that fits the first blank. The term intuitive does not support this contrast, because a visceral decision is one that is made spontaneously and without thinking, and such a decision might be driven by ones intuitive understanding of a situationa gut feeling. The part of the sentence after the semicolon makes sense, but the part of the sentence before the semicolon features a central flaw: a visceral decision is not the opposite of an intuitive judgment.

Choice (E) is incorrect. Intentional means done with a specific design, or a particular goal, in mind. Logical means related to logic and reasoning. The phrase rather than indicates that there is a contrasting relationship between the term visceral and the term that fits the first blank. The term intentional supports this contrast, because a visceral decision is made spontaneously, without thinkingit is not made with a particular goal in mind. However, the phrase that is indicates that the part of the sentence after the semicolon will restate the idea in the first part of the sentence, and this is not the case. The statement that Canady believed career choices to be logical and not rational is not a restatement of the previous idea. Further, the statement makes no sense. 5 Explanation for Correct Answer C. Choice (C) is correct. In this context, to ossify means to become fixed or inflexible. Bureaucratization describes a process of developing rules and systems that are unlikely to change. The sentence indicates that as a result of something, creative business stratagems are transformed from being versatile and adaptable to being rigid. The term bureaucratization makes sense in the second blank, because the development of rules that are unlikely to change would likely result in rigid policies. Additionally, the term ossified makes sense in the first blank, because policies that have become rigid could certainly be described as having become inflexible. Choice (A) is incorrect. To streamline something is to make it more efficient. Infighting describes rivalries and disagreements among members of a group or organization. The sentence indicates that as a result of something, creative business stratagems are transformed from being versatile and adaptable to being rigid. The term infighting does not make sense in the second blank because it does not describe a process that would necessarily result in rigid policies; disagreements might actually prevent the development of any policies. Additionally, it is somewhat illogical to suggest that infighting would lead to business stratagems being streamlined. Choice (B) is incorrect. To mitigate something is to make it less harsh or hostile. Jingoism is extreme nationalism. The sentence indicates that as a result of something, creative business stratagems are transformed from being versatile and adaptable to being rigid. The terms mitigated and jingoism do not logically complete the sentence. Nothing in the sentence suggests that the versatile and creative stratagems are harsh, so they would not likely be mitigated, or made less harsh. Additionally, extreme nationalism has nothing to do with business stratagems. Choice (D) is incorrect. To politicize something is to give it a political tone. An innovation is the introduction of something new. The sentence indicates that as a result of something, creative business stratagems are transformed from being versatile and adaptable to being rigid. The term innovation does not make sense in the second blank because it does not describe a process that would necessarily result

in rigid policies; on the contrary, innovation might lead to even more creative stratagems. The term politicized does not make sense in the first blank because nothing in the sentence suggests that the business stratagems have anything to do with politics. Choice (E) is incorrect. Venerable describes something of great age and value. Legislation is the action of making laws. The sentence indicates that as a result of something, creative business stratagems are transformed from being versatile and adaptable to being rigid. Laws can be considered rigid policies, but the word "venerable" connotes value and respect and does not fit in this context. A writer might lament the destruction of something venerable, but a writer focused on the destruction of something ("their versatility and adaptability destroyed") would not describe the result as venerable. 6 Explanation for Correct Answer D. Choice (D) is correct. The author of the passage describes the experiences of hearing wind Whistling and moaning, seeing a river of clouds, and observing a comet that looked like a feathery fishing lure and stars that winked. Meanwhile, the horses wandering nearby never glanced skywardthat is, they never seemed to notice the wondrous spectacle enjoyed by the author. When the author states that It felt good to be human, he or she suggests that the ability to appreciate the spectacle, or dramatic display, of natures beauty is uniquely human. Choice (A) is incorrect. Although the author might believe that making occasional mistakes is part of being human, he or she does not discuss this idea in this passage. Rather, the passage addresses the wondrous spectacle, or dramatic display, of nature that only humans can appreciate: the Whistling and moaning wind, the sliding river of clouds, the hanging comet, and the twinkling stars. The author expresses his or her satisfaction at having the ability to observe this display of beauty, in contrast to the horses that "never glanced skyward." Choice (B) is incorrect. The author might think that enjoying the company of others is part of being human, but he or she does not discuss this idea in this passage. The passage addresses the beauty of nature that humans, but not horses, can appreciate: the Whistling and moaning wind, the sliding river of clouds, the hanging comet, and the twinkling stars. The author seems to be enjoying this wondrous spectacle, or dramatic display, of natures beauty without the company of others. Choice (C) is incorrect. Although the passage as a whole might be considered a reflection on a past experience, the statement that It felt good to be human conveys what the author felt at the moment that he or she was experiencing the wondrous spectacle, or dramatic display, of natures beauty on Kitt Peak. The author suggests that, in contrast to the horses that "never glanced skyward," humans can appreciate the beauty of nature: the Whistling and moaning wind, the sliding river of clouds, the hanging comet, and the twinkling stars.

Choice (E) is incorrect. The author certainly seems to appreciateand might even find joy inthe things he or she observed on Kitt Peak, but the author does not address the idea of simplicity in the passage. On the contrary, the scene the author describes is somewhat complex: the wind is Whistling and moaning, the clouds look like a river, the comet hangs like a feathery fishing lure creating a gossamer swath, and the stars appear to be winking. The author suggests that appreciating such beauty in nature is part of being human and is not shared with horses or other animals. 7 Explanation for Correct Answer B. Choice (B) is correct. This passage primarily features metaphorical language, or the use of metaphors. A metaphor is a figure of speech in which the description of something is enhanced by a direct or indirect reference to something else. The author employs metaphor when he or she refers to a river of cloudsthe clouds are not really a river, but the use of the word river helps the reader visualize the scene. The author then uses similea type of metaphor that usually includes the terms like or aswhen he or she describes the comet as hanging like a feathery fishing lure. Choice (A) is incorrect. Appeal to emotion is a rhetorical device in which an author manipulates the readers emotions in order to achieve some goalfor example, persuading or convincing the reader to accept the authors argument. Although the descriptions of the scene in this passage might affect a readers emotions, there does not seem to be manipulation for any sort of purpose. Rather, the author uses descriptive language to convey his or her own emotional response to the scene. Choice (C) is incorrect. The passage does not primarily feature extended analogy, or an extended comparison of two things for the purpose of explaining an unfamiliar or difficult concept. Rather, the author uses metaphorical language to achieve an artistic effect; he or she refers to the clouds as a river, describes the comet as looking like a feathery fishing lure, and describes the stars as winking in order to help the reader visualize the scenenot to explain a concept that is difficult to understand. Choice (D) is incorrect. The entire passage relates the authors recollection of an event, but the passage itself does not feature flashbacka rhetorical device in which an earlier event is inserted into the normal chronological order of a narrative. The authors narrative is uninterrupted. Choice (E) is incorrect. Irony is a rhetorical device in which words are used to express something other than or the opposite of their literal meaning. Nothing suggests that the author of this passage is using irony. Rather, he or she seems to describe the scene on Kitt Peak with genuine appreciation. 8

Explanation for Correct Answer C. Choice (C) is correct. The passage describes Augusta Ada King as an "icon," a celebrated person to whom others are devoted. King's accomplishments are mentioned, but the real focus of the passage is on the fact that King has been celebrated above and beyond others who "have helped to advance computer science." The passage summarizes how ("She has inspired biographies, plays, novels, and even a feature film") and why ("her fascinating life and lineage") King is celebrated. Choice (A) is incorrect. The passage describes Augusta Ada King as an icon, a celebrated person to whom others are devoted. One might conclude from the passage that King was interested in computer science, but the real focus of the passage is on the fact that King has been celebrated above and beyond others who have helped to advance computer science. The passage does not directly address Kings interest in computer science. Choice (B) is incorrect. The passage describes Augusta Ada King as an "icon," a celebrated person to whom others are devoted. Although the passage mentions Kings fascinating life and lineage, it does not provide a character analysis. The focus of the passage is on the fact that King has been celebrated above and beyond others who "have helped to advance computer science." Choice (D) is incorrect. The passage describes Augusta Ada King as an "icon," a celebrated person to whom others are devoted. King's accomplishments in computer science are mentioned, but the real focus of the passage is on the fact that King has been celebrated above and beyond others who "have helped to advance computer science." The passage does not trace the development of the modern-day computer. Choice (E) is incorrect. The passage describes Augusta Ada King as an "icon," a celebrated person to whom others are devoted. The focus of the passage is on the fact that King has been celebrated above and beyond others who "have helped to advance computer science." King's example might have inspired other women to become computer scientists, but the passage is not primarily concerned with encouraging more women to pursue careers in computer science. 9 Explanation for Correct Answer A. Choice (A) is correct. The author of the passage would surely disagree with the statement that King's family history plays no part in her fame. The author states unambiguously that King's "lineage (she was the daughter of the flamboyant poet Lord Byron)" is a factor in her status as an "icon," or celebrated person to whom others are devoted. The fact that King had a famous father is clearly significant.

Choice (B) is incorrect. The author indicates that King "published an influential set of notes describing . . . the first design for an automatic computer." Her notes also included her "program for computing a series of figures called Bernoulli numbers." King clearly made important and original contributions to computer science, so it is likely that the author would agree, not disagree, with the statement in (B). Choice (C) is incorrect. The author indicates that King "has inspired biographies, plays, novels, and even a feature film." Interest in King appears to have spread throughout popular culture, so the author would likely agree, not disagree, with the statement in (C). Choice (D) is incorrect. The author states that "King has had a computer language named after her," which clearly signifies King's lasting prominence in the field of computer science. The author would likely agree, not disagree, with the statement that King was well known in the field of computer science long after she had completed her work. Choice (E) is incorrect. The author refers to King's "fascinating life and lineage" as one reason that she became an "icon," or celebrated person to whom others are devoted. The other reason is "her role as a female pioneer in her field" of computer science. This suggests that King's life would have been noteworthy even if she had not been a computer scientist. The author likely would not disagree with the statement in (E). 10 Explanation for Correct Answer D. Choice (D) is correct. The author of the passage claims that Black Americans have managed to sustain links with the continent of their origin, keeping various ties to Africa intact, and indicates that Black Americans and Africans are linked. People in these two groups may believe that the roots that had once bound [them] together had already withered and died, but they nevertheless have an ongoing relationship; the author writes that as long as Black people in America have reached back to Africa . . . Africans have reached to Black people in the Americas. The primary purpose of the passage is to emphasize the significance of the ongoing relationship between Black Americans and Africans. Choice (A) is incorrect. The author does describe the relationship between Black Americans and Africans: the two groups are connected through their ties to the African continent. Although the author does briefly mention that Black people in America have reached back to Africa to offer . . . reassurance, reaffirmation, fraternity, and strength, his primary purpose does not seem to be to show the impact Black Americans have had on African societies. If anything, he emphasizes the impact African societies have had on Black Americans, noting African proverbs and stating that Africa has left her mark on all of us.

Choice (B) is incorrect. The author does describe the relationship between Black Americans and Africans: the two groups are connected through their ties to the African continent. However, nowhere in the passage does the author indicate that Africans are attempting to embrace American culture. Rather, the proverbs he describes in the first paragraph and his statement that Africa has left her mark on all of us suggest that Americans are affected by and can learn from African culture, not that Africans are eagerly adopting aspects of American culture. Choice (C) is incorrect. Ambivalence describes simultaneous contradictory feelings toward an object or person. The author of the passage does briefly mention in the last paragraph that when Black Americans and Africans finally [meet], there is sometimes disappointment because a feeling of kinship might not be immediate; however, the passage does not support the idea that one community has both positive and negative feelings for the other. Rather, the author emphasizes that Through ingenuity and dogged determination . . . Black Americans have kept various ties to Africa intact and that Africans have reached to Black people in the Americas. The author primarily focuses on the positive relationship between the two groups, noting that when they have made contact, the world has been forced to take note. Choice (E) is incorrect. The author does describe the relationship between Black Americans and Africans, but the ties are not really cultural, or based on intellectual or artistic commonality. The author speaks of Black Americans reach[ing] back to Africa to offer and receive reassurance, which seems to emphasize emotional rather than cultural ties. The author does describe two proverbs, both of which are aspects of Kikongo culture. However, he does not indicate that Black Americans and Africans share these proverbs within their respective cultures. The main emphasis of the passage is not on cultural ties between two peoplesand not between two nations. 11 Explanation for Correct Answer B. Choice (B) is correct. The message that a people cannot flourish without their lifegiving foundations in the past is best characterized as cautionary advice, or a kind of warning. The author states that his ancestors felt that connections with the past are essential if a people is to survivein other words, the ancestors warned that the survival of a community depends on its ties to its roots. Choice (A) is incorrect. The author states that the Kikongo proverb sends a clear and powerful message from his ancestors, not a veiled, or somewhat disguised, criticism. The author writes that the message explicitly emphasizes that the survival of a community depends on its ties to its roots. Choice (C) is incorrect. The author states that the Kikongo proverb sends a message from his ancestors; nothing in the passage suggests that the message is a questionable proposition, or a potentially problematic proposal. Rather, the message

gives a sort of warning, stating that the survival of a community depends on its ties to its roots. Choice (D) is incorrect. Although the author might feel nostalgic, or sentimental, when recalling his ancestors proverb, the message is an interpretation of the proverb, not a description of what the author is feeling. The message gives a sort of warning, stating that the survival of a community depends on its ties to its roots. Choice (E) is incorrect. Although the Kikongo proverb might be said to contain a predictionthat people who do not maintain connections to the past will not survive or flourishthe prediction is certainly not optimistic. The message of the proverb is a warning: the survival of a community depends on its ties to its roots. 12 Explanation for Correct Answer C. Choice (C) is correct. An inherent characteristic is one that is firmly fixed in a person; it is not changing. In the sentence after the proverb, the author writes that the past has a seemingly instinctive pull on people because of their inborn curiosity. The use of the terms instinctive and inborn suggest that interest in ones history is fixed before birth. Therefore, inherent interest in their history is a logical way to describe the curiosity people have about their origins. Choice (A) is incorrect. In the sentence after the proverb, the author writes that the past has a seemingly instinctive pull on people because of their inborn curiosity. This sentence also indicates that the proverb in lines 15-16 is a metaphor, not a literal discussion of children. Further, nothing in the rest of the passage suggests that the author is describing the actual behavior of young children. This proverb offers information about the inherent interest people have in their history. Choice (B) is incorrect. The proverb in lines 15-16 does describe one way in which a child is vulnerable: it may be stolen from its parents. However, nothing in the rest of the passage suggests that the author is describing actual incidents that children experience. In the sentence after the proverb, the author writes that the past has a seemingly instinctive pull on people because of their inborn curiosity. The sentence indicates that the proverb emphasizes the inherent interest people have in their history, not the vulnerability of children. Choice (D) is incorrect. The proverb in lines 15-16 does describe a situation that a family might experience. However, nothing in the rest of the passage suggests that the author relates the proverb to demonstrate the complexity of familial relations. In the sentence after the proverb, the author writes that the past has a seemingly instinctive pull on people because of their inborn curiosity. The sentence indicates that the proverb emphasizes the inherent interest people have in their history, not the complexity of familial relations.

Choice (E) is incorrect. The proverb in lines 15-16 could be an example of a family being undermined. However, the author does not address the consequences of stealing a child. Rather, in the sentence after the proverb, the author writes that the past has a seemingly instinctive pull on people because of their inborn curiosity. The sentence indicates that the proverb emphasizes the inherent interest people have in their history and is not intended to warn those who seek to undermine the family. 13 Explanation for Correct Answer E. Choice (E) is correct. If something is unsubstantiated, it is not proven by evidence. In line 41, the author uses the term shadowy to describe imaginings, or impressions about something that is not actually happening; these impressions are indistinct and unverifiable. The fact that the imaginings do not usually hold up in the light of real experience further suggests that they are unsubstantiated. Choice (A) is incorrect. In line 41, the author uses the term shadowy to describe imaginings, or impressions about something that is not actually happening. There is nothing in the sentence to suggest that these imaginings are gloomy, or dark or sad. Rather, the Shadowy imaginings are thoughts that do not usually hold up in the light of real experience; they are unsubstantiatednot proven by evidence but not necessarily gloomy. Choice (B) is incorrect. In line 41, the author uses the term shadowy to describe imaginings, or impressions about something that is not actually happening. Because imaginings are simply ideas a person has rather than real events, they might be secret, but nothing in the sentence suggests that the author describes them as shadowy because he wishes to conceal or hide them. In lines 41-42, the contrast between Shadowy imaginings and real experience indicates that these imaginings are unsubstantiated, or not proven by evidence, rather than secret. Choice (C) is incorrect. If something is sinister, it is evil or wrong. In line 41, the author uses the term shadowy to describe imaginings, or impressions about something that is not actually happening. Nothing in the sentence suggests that these imaginings are evil. Rather, Shadowy imaginings are thoughts that do not usually hold up in the light of real experience; therefore, they are unsubstantiated, or not proven by evidence, not sinister. Choice (D) is incorrect. In line 41, the author uses the term shadowy to describe imaginings, or impressions about something that is not actually happening. Because imaginings are simply ideas a person has rather than real events, they might be concealed, or hidden, but nothing in the sentence suggests that the author wishes to hide them. In lines 41-42, the contrast between Shadowy imaginings and real experience indicates that these imaginings are unsubstantiated, or not proven by evidence, rather than concealed. 14

Explanation for Correct Answer E. Choice (E) is correct. In the sentence that starts in line 42, the author asks if [Black Americans and Africans] hadnt been mistaken that kinship exists between the two groups, thus indicating that there have been feelings of uncertainty or doubt. But he goes on to state that despite that initial doubt, the two groups are time and again . . . reminded of what [is] shared. Finally, in the last sentence, the author remarks that when Africans and Black Americans have reached out to one another and formed relationships through literature, politics, music, and religion . . . the world has been forced to take note. The author clearly feels proud of the relationshipsand their effectsthat Black Americans and Africans have formed. The sentences in lines 4250 illustrate a shift from doubt to pride. Choice (A) is incorrect. In the sentence that starts in line 42, the author shows doubt, wondering if we hadnt been mistaken that ties between Africans and Black Americans exist. Although doubt may be accompanied by feelings of fear, nothing in this paragraph suggests that the author is or was afraid. In the last sentence, the author notes that when Africans and Black Americans form relationships through literature, politics, music, and religion . . . the world has been forced to take note. There is no indication that members of the two groups must show special courage in order to form these relationships, of which the author is clearly proud. Choice (B) is incorrect. In the sentence that starts in line 42, the author shows doubt, wondering if we hadnt been mistaken that ties between Africans and Black Americans exist. Although doubt may be accompanied by feelings of anger, nothing in this last paragraph suggests that the author is angry. Moreover, because the two groups are not especially angry with one another, their kinship does not require forgiveness. Choice (C) is incorrect. In the sentence that starts in line 42, the author asks if we hadnt been mistaken that ties between Africans and Black Americans exist, indicating uncertainty, or doubt. However, the tone of the last sentence does not reflect despair, or a loss of hope. Rather, in the last sentence, the author displays hope and pride rather than hopelessness, claiming that the world has been forced to take note of the relationships between Africans and Black Americans. Choice (D) is incorrect. In the sentence that starts in line 42, the author asks if we hadnt been mistaken that ties between Africans and Black Americans exist, indicating uncertainty, or doubt. The author suggests that one cause of this uncertainty is the institution of slavery, which may produce a tone of regret, or a feeling of sorrow about the past. However, the tone of the last sentence is not one of determination, or resolution to achieve a desired result. The author is expressing pride in past relationships between the two groups (through literature, politics, music, and religion . . . the world has been forced to take note) rather than a determination to change those relationships in the future.

15 Explanation for Correct Answer B. Choice (B) is correct. The author makes many broad generalizations, statements that apply to or describe a wide range of people or situations. The author does not use many specifics to support his point that Africans and Black Americans are connected by their links to Africa. Rather, in the first paragraph, he describes the desire to learn about ones past as a quest we all share, and later in the passage he writes, Africa has left her mark on all of us. Through these statements the author describes broad characteristics, feelings, and attitudes of large groups of people. Choice (A) is incorrect. The authors point in this passage is that despite the distance (across time and space) between Africans and Black Americans, they are still connected by their links to Africa. He supports his point through claims that the desire to learn about ones past is a quest we all share, and that Africa has left her mark on all of us. Hypothetical scenarios are accounts of events that have not happened or are imagined. The author describes specific events that have happened ("whenever we've made contact, the world has been forced to take note") in a vague, general way, conveying his point through the use of broad generalizations, not hypothetical scenarios. Choice (C) is incorrect. The authors point in this passage is that despite the distance (across time and space) between Africans and Black Americans, they are still connected by their links to Africa. He conveys this point through Kikongo proverbs and broad generalizations. He does not primarily rely on historical facts. A statement like "whenever we've made contact, the world has been forced to take note" may be based on historical fact, but it is better described as a broad generalization. Choice (D) is incorrect. The authors point in this passage is that despite the distance (across time and space) between Africans and Black Americans, they are still connected by their links to Africa. He supports his point with Kikongo proverbs and broad generalizations, both of which describe the characteristics of large groups of people. Therefore, he conveys his point through the use of broad generalizations, not stories about himself, or personal anecdotes. One reflection of this strategy is the authors repeated use of the pronoun "we," which suggests generalizations about a group of people, in place of "I," which would indicate personal anecdotes. Choice (E) is incorrect. The authors point in this passage is that despite the distance (across time and space) between Africans and Black Americans, they are still connected by their links to Africa. If the author were making a scholarly analysis, or a well-supported examination of an idea, he would need to convey his point using the work of other scholars. Instead, he conveys his point through Kikongo proverbs and other generalizations. 16

Explanation for Correct Answer C. Choice (C) is correct. Both passages refer to the enormous popular appeal of the painting: Passage 1 mentions that "1.6 million people jostled to see it" when it was shown in New York, and Passage 2 mentions that millions of people "know the name of only one picturethe Mona Lisa." Further, both passages attempt to explain why the painting has such appeal. The question of the Mona Lisa's popular appeal is a central concern of both passages. Choice (A) is incorrect. While Passage 1 alludes to the Mona Lisa's "famous smile," Passage 2 does not mention that particular feature of the painting. Choice (B) is incorrect. Passage 1 addresses the question of the Mona Lisa's identity ("its subject was nobody special, probably the wife of a Florentine merchant named Giocondo"), but Passage 2 does not. Choice (D) is incorrect. Passage 1 indicates the various ways in which da Vinci's painting influenced later artists: the use of perspective and triangular composition; the relaxed pose; the technique of starting with dark undertones and adding "layers and layers of thin, transparent glazes." Passage 2, on the other hand, is not concerned with the painting's influence on artists. Passage 2 is concerned with the painting's impact on people in general, even "those who have no taste or passion for painting."

Choice (E) is incorrect. Neither passage indicates that the painting is deteriorating or has suffered any type of damage or ill effects. Rather, it appears that the painting, despite its age, still has substantial power to influence those who view it. 17 Explanation for Correct Answer A. Choice (A) is correct. The author of Passage 2 says that, in attempting to explain the great appeal of the Mona Lisa, he or she starts with the assumption that the renown of a masterpiece rests on a sequence of events and historical agencies (people, institutions, processes) working in a largely unplanned manner for different ends." In other words, the Mona Lisa's renown, or fame, is a result of many different actions and decisions over the yearsevents such as the decision of Napoleon to hang the painting in his bedroom, or the decision of a New York museum to hold a special showing of the painting, or the decision of a Tokyo museum to allow viewers only ten seconds each to view the painting. Choice (B) is incorrect. The author of Passage 2 believes that the true importance of the Mona Lisa, or of any masterpiece, "rests on a sequence of events and historical agencies (people, institutions, processes) working in a largely unplanned manner for

different ends." The events listed in the first paragraph of Passage 1 are examples of such events. These events, then, do not "distort" the true importance of the Mona Lisa; rather, they give the Mona Lisa whatever importance it has. Choice (C) is incorrect. There is no indication that the author of Passage 2 would consider the events listed in Passage 1 likely to cause art enthusiasts any annoyance whatsoever. The author of Passage 2 mentions art enthusiasts in line 34"Art historians, poets, and admirers have tried to explain the commanding place that the Mona Lisa has"but does not predict how these individuals might respond to events such as those mentioned in the first paragraph of Passage 1. Choice (D) is incorrect. The author of Passage 2 believes that the "renown," or fame, of any masterpiece "rests on a sequence of events and historical agencies (people, institutions, processes) working in a largely unplanned manner for different ends." The events listed in the first paragraph of Passage 1 are examples of such events. These events, then, are worthy of the consideration of art historians, and possibly art critics as well. Choice (E) is incorrect. The author of Passage 2 explains that, in attempting to explain the great appeal of the Mona Lisa, "like most historians, he or she starts with the assumption that the renown of a masterpiece rests on a sequence of events and historical agencies (people, institutions, processes) working in a largely unplanned manner for different ends." The events listed in the first paragraph of Passage 1 are examples of such events. These events, then, would seem to be convenient, not inconvenient, facts for many art historians. 18 Explanation for Correct Answer E. Choice (E) is correct. The word "But" at the beginning of the second sentence of the paragraph indicates a contrast with the previous sentence. While the first sentence notes that the subject of the Mona Lisa was "nobody special," the second sentence counters that "her portrait set the standard for High Renaissance paintings." In other words, an unimportant (ordinary) woman is the subject of an important (aesthetically significant) painting. Choice (A) is incorrect. The word "But" at the beginning of the second sentence of the paragraph indicates a contrast with the previous sentence. While the first sentence notes that the subject of the Mona Lisa was "nobody special," the second sentence counters that "her portrait set the standard for High Renaissance paintings." The first sentence indicates nothing about the woman's appearance, only that she was "probably the wife of a Florentine merchant." Choice (B) is incorrect. The word "But" at the beginning of the second sentence of the paragraph indicates a contrast with the previous sentence. While the first sentence notes that the subject of the Mona Lisa was "nobody special," the second sentence counters that "her portrait set the standard for High Renaissance paintings."

The woman may have been from "humble origins," or a poor background, but the second sentence indicates nothing about her portrait's monetary value. Choice (C) is incorrect. The word "But" at the beginning of the second sentence of the paragraph indicates a contrast with the previous sentence. While the first sentence notes that the subject of the Mona Lisa was "nobody special," the second sentence counters that "her portrait set the standard for High Renaissance paintings." The first sentence does not indicate that the woman suffered an "untimely demise," or early death, and the second sentence does not discuss her portraits immortality. Choice (D) is incorrect. The word "But" at the beginning of the second sentence of the paragraph indicates a contrast with the previous sentence. While the first sentence notes that the subject of the Mona Lisa was "nobody special," the second sentence counters that "her portrait set the standard for High Renaissance paintings." The first sentence does not indicate that the woman exhibited a lack of charisma, or charm. The phrase "nobody special" refers to her social status, as the information about her husband suggestsnot to her personal characteristics. 19 Explanation for Correct Answer B. Choice (B) is correct. The third paragraph of Passage 1 discusses the characteristics of the Mona Lisa. The author explains that Leonardo built the illusion of threedimensional features through layers and layers of thin, transparent glazes. . . . [creating] a continuous gradation of subtle tones, without crisp separating edges. As Leonardos quote indicates, the artist used a special painting technique to create a unique effect: his painting appeared to be without lines or borders, in the manner of smoke. Choice (A) is incorrect. The third paragraph of Passage 1 discusses the characteristics of the Mona Lisa. The author explains that Leonardo "built the illusion of three-dimensional features through layers and layers of thin, transparent glazes. . . . [creating] a continuous gradation of subtle tones, without crisp separating edges." As Leonardo's quote indicates, the artist used a special painting technique to create a unique effect: his painting appeared to be "without lines or borders, in the manner of smoke." Leonardos quote serves to characterize an effect, not to defend a methodology. Choice (C) is incorrect. The third paragraph of Passage 1 discusses the characteristics of the Mona Lisa. The author explains that Leonardo "built the illusion of three-dimensional features through layers and layers of thin, transparent glazes. . . . [creating] a continuous gradation of subtle tones, without crisp separating edges." As Leonardo's quote indicates, the artist used a special painting technique to create a unique effect: his painting appeared to be "without lines or borders, in the manner of smoke." The author is not criticizing Leonardo's technique, and Leonardo is not criticizing his own technique or any other artists technique. Instead, Leonardos quote serves to characterize an effect.

Choice (D) is incorrect. The third paragraph of Passage 1 discusses the characteristics of the Mona Lisa. The author explains that Leonardo "built the illusion of three-dimensional features through layers and layers of thin, transparent glazes. . . . [creating] a continuous gradation of subtle tones, without crisp separating edges." As Leonardo's quote indicates, the artist used a special painting technique to create a unique effect: his painting appeared to be "without lines or borders, in the manner of smoke." The author is not downplaying Leonardo's accomplishment, and Leonardo is not downplaying his own accomplishments or those of any other artist. Instead, Leonardos quote serves to characterize an effect. Choice (E) is incorrect. The third paragraph of Passage 1 discusses the characteristics of the Mona Lisa. The author explains that Leonardo "built the illusion of three-dimensional features through layers and layers of thin, transparent glazes. . . . [creating] a continuous gradation of subtle tones, without crisp separating edges." Leonardo rejected tradition ("Instead of proceeding from outlined figures, as painters did before . . .") and used a new technique to create a unique effect. Leonardos quote does not serve to acknowledge an influence; rather, Leonardos statement that his paintings appeared in the manner of smoke serves to characterize an effect. 20 Explanation for Correct Answer A. Choice (A) is correct. The reference at the end of Passage 1 to "that famous smile" suggests that one aspect of the subject of the Mona Lisa that has attracted attention is her mouth. In line 32, the author of Passage 2 indicates that Mona Lisas eyes and hands are instantly recognizable to many people. It seems clear from the reference in Passage 1 that her mouth is recognizable as well. Choice (B) is incorrect. In line 32, the author of Passage 2 indicates that the eyes and hands of the subject of the Mona Lisa are instantly recognizable to many people. It seems clear from the reference to "that famous smile" in Passage 1 that her mouth is recognizable as well. Mona Lisa's hair is not mentioned in Passage 1. Choice (C) is incorrect. In line 32, the author of Passage 2 indicates that the eyes and hands of the subject of the Mona Lisa are instantly recognizable to many people. It seems clear from the reference to "that famous smile" in Passage 1 that her mouth is recognizable as well. Mona Lisa's nose is not mentioned in Passage 1. Choice (D) is incorrect. In line 32, the author of Passage 2 indicates that the eyes and hands of the subject of the Mona Lisa are instantly recognizable to many people. It seems clear from the reference to "that famous smile" in Passage 1 that her mouth is recognizable as well. Mona Lisa's chin is not mentioned in Passage 1. Choice (E) is incorrect. In line 32, the author of Passage 2 indicates that the eyes and hands of the subject of the Mona Lisa are instantly recognizable to many people. It seems clear from the reference to "that famous smile" in Passage 1 that

her mouth is recognizable as well. Mona Lisa's profile is mentioned indirectly in Passage 1, in that she appears "in a relaxed, natural, three-quarter pose" rather than in the position of one of the "stiff, profile portraits that had been the norm." But this suggests that Mona Lisa's profilea stiff side viewdoes not appear in the painting. 21 Explanation for Correct Answer D. Choice (D) is correct. The first paragraph of Passage 2 opens with the question, "Why is the Mona Lisa the best-known painting in the entire world?" and provides an answer, citing "Art historians, poets, and admirers": "There is something, they argue, inside the painting that speaks to us all." This point of view, as the last sentence of the paragraph indicates, "is still the position of many art critics." In this context, the term "position" means view, or point of view. Choice (A) is incorrect. The first paragraph of Passage 2 opens with the question, "Why is the Mona Lisa the best-known painting in the entire world?" and provides an answer, citing "Art historians, poets, and admirers": "There is something, they argue, inside the painting that speaks to us all." This point of view, as the last sentence of the paragraph indicates, "is still the position of many art critics." In this context, the term "position" means view, or point of view, not rank. Choice (B) is incorrect. The first paragraph of Passage 2 opens with the question, "Why is the Mona Lisa the best-known painting in the entire world?" and provides an answer, citing "Art historians, poets, and admirers": "There is something, they argue, inside the painting that speaks to us all." This point of view, as the last sentence of the paragraph indicates, "is still the position of many art critics." In this context, the term "position" means view, or point of view, not role. Choice (C) is incorrect. The first paragraph of Passage 2 opens with the question, "Why is the Mona Lisa the best-known painting in the entire world?" and provides an answer, citing "Art historians, poets, and admirers": "There is something, they argue, inside the painting that speaks to us all." This point of view, as the last sentence of the paragraph indicates, "is still the position of many art critics." In this context, the term "position" means view, or point of view, not policy. Choice (E) is incorrect. The first paragraph of Passage 2 opens with the question, "Why is the Mona Lisa the best-known painting in the entire world?" and provides an answer, citing "Art historians, poets, and admirers": "There is something, they argue, inside the painting that speaks to us all." This point of view, as the last sentence of the paragraph indicates, "is still the position of many art critics." In this context, the term "position" means view, or point of view, not location. 22 Explanation for Correct Answer E.

Choice (E) is correct. Paul Barolsky attributes the power of the Mona Lisa to "Leonardo's remarkable technique, which creates a sense of texture and depth," and adds that the painting conveys the "mind or soul" of the woman it portrays. Barolsky's use of the word "depth" indicates that the two-dimensional image appears to occupy three dimensions: height, width, and depth. The author of Passage 1 agrees that Leonardo's technique allowed the artist to create "the illusion of threedimensional features." Choice (A) is incorrect. Paul Barolsky attributes the power of the Mona Lisa to "Leonardo's remarkable technique, which creates a sense of texture and depth," and adds that the painting conveys the "mind or soul" of the woman it portrays. Neither Barolsky nor the author of Passage 1 states that the painting elicits idiosyncratic, or unique and peculiar, responses from viewers; on the contrary, both suggest that many people have similar responses to the painting. Choice (B) is incorrect. Paul Barolsky attributes the power of the Mona Lisa to "Leonardo's remarkable technique, which creates a sense of texture and depth," and adds that the painting conveys the "mind or soul" of the woman it portrays. Both Barolsky and the author of Passage 1 indicate that the painting is revered, or highly respected, by many people, but neither suggests that it is revered "unduly," or excessively. Choice (C) is incorrect. Paul Barolsky attributes the power of the Mona Lisa to "Leonardo's remarkable technique, which creates a sense of texture and depth," and adds that the painting conveys the "mind or soul" of the woman it portrays. The author of Passage 1 states that the painting has influenced many artists, but Barolsky does not make this point. Choice (D) is incorrect. Paul Barolsky attributes the power of the Mona Lisa to "Leonardo's remarkable technique, which creates a sense of texture and depth," and adds that the painting conveys the "mind or soul" of the woman it portrays. The author of Passage 1 states that the painting was intended to be framed and hung, but Barolsky indicates nothing about how the painting was intended to be displayed. 23 Explanation for Correct Answer E. Choice (E) is correct. At the beginning of the last paragraph, the author of Passage 2 explains that "one should avoid succumbing to the charm of a myth." The "myth" is the idea that there is something inside each famous work of art that speaks directly to every viewer, independent of the work's history or reputation. The author is clearly skeptical of this idea, believing that "the renown of a masterpiece rests on a sequence of events and historical agencies" rather than on a "mysterious yet almost palpable contact" with each viewer's "sensibility." By putting quotation marks around the word "innate," the author indicates this skepticism, suggesting that the sensibility that leads a viewer to appreciate a famous work of art is not really innate, or something a person is born with. In other words, according to the author, viewers

do not admire paintings like the Mona Lisa solely because of a "mysterious" connection between the painting and the viewer's innate, inborn sensibility; they admire them, at least in part, because the paintings are famous. Choice (A) is incorrect. At the beginning of the last paragraph, the author of Passage 2 explains that "one should avoid succumbing to the charm of a myth." The "myth" is the idea that there is something inside each famous work of art that speaks directly to every viewer, independent of the work's history or reputation. The author is clearly skeptical of this idea, believing that "the renown of a masterpiece rests on a sequence of events and historical agencies" rather than on a "mysterious yet almost palpable contact" with each viewer's "sensibility." By putting quotation marks around the word "innate," the author indicates this skepticism. There is no suggestion that the sensibility discussed is part of a revolutionary movement. Choice (B) is incorrect. At the beginning of the last paragraph, the author of Passage 2 explains that "one should avoid succumbing to the charm of a myth." The "myth" is the idea that there is something inside each famous work of art that speaks directly to every viewer, independent of the work's history or reputation. The author is clearly skeptical of this idea, believing that "the renown of a masterpiece rests on a sequence of events and historical agencies" rather than on a "mysterious yet almost palpable contact" with each viewer's "sensibility." By putting quotation marks around the word "innate," the author indicates this skepticism. There is no suggestion that the sensibility discussed is an overused artistic technique. Choice (C) is incorrect. At the beginning of the last paragraph, the author of Passage 2 explains that "one should avoid succumbing to the charm of a myth." The "myth" is the idea that there is something inside each famous work of art that speaks directly to every viewer, independent of the work's history or reputation. The author is clearly skeptical of this idea, believing that "the renown of a masterpiece rests on a sequence of events and historical agencies" rather than on a "mysterious yet almost palpable contact" with each viewer's "sensibility." By putting quotation marks around the word "innate," the author indicates this skepticism. There is no hint that the term "innate," which means inborn, is being used symbolically. Choice (D) is incorrect. At the beginning of the last paragraph, the author of Passage 2 explains that "one should avoid succumbing to the charm of a myth." The "myth" is the idea that there is something inside each famous work of art that speaks directly to every viewer, independent of the work's history or reputation. The author is clearly skeptical of this idea, believing that "the renown of a masterpiece rests on a sequence of events and historical agencies" rather than on a "mysterious yet almost palpable contact" with each viewer's "sensibility." By putting quotation marks around the word "innate," the author indicates this skepticism, suggesting that the sensibility that leads a viewer to appreciate a famous work of art is not really "innate," or something a person is born with. There is no hint that the term "innate" is being highlighted for any reason other than this. 24

Explanation for Correct Answer C. Choice (C) is correct. Both passages discuss the Mona Lisa, but focus on very different aspects of the painting. The author of Passage 1 stresses that the painting "set the standard for High Renaissance paintings in many important ways," and then explains what those ways were, highlighting Leonardo's stylistic innovations. The author of Passage 2, on the other hand, opens with the question, "Why is the Mona Lisa the best-known painting in the entire world?" and, after discussing the role of "qualities intrinsic to the work," ultimately concludes that one should "start with the assumption that the renown of a masterpiece rests on a sequence of events and historical agencies." The author of Passage 2 is seeking to account for the fame, or cultural preeminence, of the Mona Lisa. Choice (A) is incorrect. The author of Passage 1 does not stress the unique smile in the Mona Lisa; he or she only mentions the smile briefly at the end of the passage. Further, the author of Passage 2 does not focus on the woman who was the subject of the painting; he or she focuses on how the painting is received by viewers. Choice (B) is incorrect. The author of Passage 1 does explain the techniques Leonardo used to produce the striking appearance of the Mona Lisa, but the author of Passage 2 does not examine Leonardo's background. Leonardo's background is not mentioned anywhere in Passage 2. Choice (D) is incorrect. The author of Passage 1 does speculate briefly about the life of the woman who was the subject of the painting: she was "probably the wife of a Florentine merchant." However, the author of Passage 2 does not argue that historical interpretations are irrelevant. He or she simply states that, "like most historians, I start with the assumption that the renown of a masterpiece rests on a sequence of events and historical agencies." The author of Passage 2 favors historical interpretations over aesthetic ones. Choice (E) is incorrect. The author of Passage 1 does not explicitly allude to the societal importance of the Mona Lisa, and the author of Passage 2 does not explicitly debate the paintings artistic merits. The author of Passage 1 does indicate that the painting has great artistic importance and is extremely popular, and the author of Passage 2 implies that the Mona Lisa may enjoy continued fame for reasons other than its intrinsic qualities. But the passages are not effectively characterized by the statement in (E). Section #5: View Explanations 1 Explanation for Correct Answer C. Choice (C) is correct. Hospitality is characterized by the generous and pleasant reception of guests. If Miranda eagerly welcomes anyone into her home, it makes sense that she would be known for her hospitality.

Choice (A) is incorrect. Cowardice is a lack of courage or resolution. The structure of the sentence suggests that the word that fits the blank will describe Mirandas eagerness to welcome people into her home. Demonstrating a lack of courage is not related to eagerly welcoming guests, so the term cowardice does not logically complete the sentence. Instead, the term hospitality best describes Mirandas eagerness. Choice (B) is incorrect. Prudence is characterized by good judgment or shrewdness in the management of affairs. The structure of the sentence suggests that the word that fits the blank will describe Mirandas eagerness to welcome people into her home. Demonstrating good judgment while managing something is not related to eagerly welcoming guests, so the term prudence does not logically complete the sentence. Instead, the term hospitality best describes Mirandas eagerness. Choice (D) is incorrect. Aloofness is the quality of being removed or distant, either emotionally or physically. The structure of the sentence suggests that the word that fits the blank will describe Mirandas eagerness to welcome people into her home. It does not make sense to describe someone who eagerly welcomes guests as aloof, or emotionally or physically distant. Rather, Miranda would likely be described as demonstrating hospitality. Choice (E) is incorrect. Loyalty is the quality of being faithful to a person, cause, institution, or product. The structure of the sentence suggests that the word that fits the blank will describe Mirandas eagerness to welcome people into her home. Someone who is loyal might be enthusiastic to welcome certain people into her home, but there is no connection between demonstrating loyalty and eagerly welcoming "anyone," so the term loyalty is not the best choice. Rather, the term hospitality best describes Mirandas eagerness. 2 Explanation for Correct Answer B. Choice (B) is correct. To derail something is to obstruct its progress. The court decision disappointed those who supported the governors plan, so the court must not have been in favor of the plan. It makes sense to suggest that those who wanted to set aside land for a forest preserve would be disappointed when the plan was derailed, or obstructed. Choice (A) is incorrect. To applaud something is to express approval of it or to praise it. It does not make sense to suggest that the supporters of the governors plan would be disappointed by a court decision that applauded, or approved of, that plan. On the contrary, one would expect that the supporters would be pleased with such a decision. Choice (C) is incorrect. In this context, to acknowledge something is to take notice of it. The sentence states that the court decision disappointed the people who

supported the governors plan, so the court must not have been in favor of the plan. There is no reason to expect that the plan's supporters would be disappointed by a decision that acknowledged, or took notice of, the plan, unless more information regarding the decision is provided. Choice (D) is incorrect. To permit something is to allow it. It does not make sense to suggest that the supporters of the governors plan would be disappointed by a court decision that permitted, or allowed, that plan to go into effect. On the contrary, one would expect that the supporters would be pleased with such a decision. Choice (E) is incorrect. In this context, to anticipate something is to foresee it or to give it thought in advance. The sentence states that the court decision disappointed the people who supported the governors plan, so the court must not have been in favor of the plan. There is no reason to expect that the plan's supporters would be disappointed by a decision that anticipated, or foresaw, the plan, unless more information about the decision is provided. 3 Explanation for Correct Answer C. Choice (C) is correct. A stimulus is something that stimulates, rouses, or incites to activity. The sentence states that brain activity increases when one plays a musical instrument; therefore, it makes sense to describe "playing a musical instrument" as a "stimulus to promote learning." Choice (A) is incorrect. A condition is something essential to the appearance or occurrence of something else. The sentence states that brain activity increases when one plays a musical instrument; therefore, playing an instrument can promote learning. It does not make sense to refer to the act of playing a musical instrument as a condition for learning because children can learn and increase their brain activity in other ways. Playing a musical instrument is not essential to the occurrence of learning. Choice (B) is incorrect. A highlight is an event or detail that is of major significance or special interest. Although something that increases brain activity could certainly be described as being of major significance, it does not make sense to describe the act of playing a musical instrument as a detail of something larger, let alone a detail that can be used to promote learning. Choice (D) is incorrect. A dictum is a noteworthy statement or announcement. The sentence states that brain activity increases when one plays a musical instrument; therefore, playing an instrument can promote learning. It does not make sense to describe the act of playing a musical instrument as a dictum, or an announcement, to promote learning.

Choice (E) is incorrect. A respite is an interval of rest or relief. The sentence states that brain activity increases when one plays a musical instrument; therefore, playing an instrument can promote learning. It is somewhat illogical to describe an engaging activitysuch as playing a musical instrumentas a respite, or interval of rest. Additionally, the sentence does not indicate why the children would need rest or relief. 4 Explanation for Correct Answer A. Choice (A) is correct. To negotiate is to confer with another in order to settle some matter. In this context, to make a concession is to yield something, or to give something up. The sentence indicates that although two different attitudes will be described, the difference between these attitudes will be subtle; this suggests that the two attitudes will actually be similar in some way. The terms negotiate and concessions logically complete the sentence. Although concessions are frequently made during negotiations, being willing to confer to settle a matter is not the same as being obviously ready to give things up; these attitudes are related but different. Choice (B) is incorrect. To antagonize is to incur or provoke hostility. To make friends is to become friends with someone. The sentence indicates that two different attitudes will be described. However, the difference between these attitudes will be subtle; this suggests that the two attitudes will actually be similar in some way. The terms antagonize and friends do not logically complete the sentence. A countrys willingness to provoke hostility is not related to its readiness to become friendly with another country. Choice (C) is incorrect. To surrender is to yield to the power, control, or possession of another. To make enemies is to cause others to be hostile. The sentence indicates that two different attitudes will be described. However, the difference between these attitudes will be subtle; this suggests that the two attitudes will actually be similar in some way. The terms surrender and enemies do not logically complete the sentence. A countrys willingness to yield to the power of another country is not related to its readiness to make enemies. Choice (D) is incorrect. To "dominate" is to rule or control. "Inquiries" are requests for information. The sentence indicates that two different attitudes will be described. However, the difference between these attitudes will be subtle; this suggests that the two attitudes will actually be similar in some way. The terms dominate and inquiries do not logically complete the sentence. There is no direct connection between a countrys willingness to rule and its readiness to request information; therefore, it does not make sense to say that the difference between the attitudes is subtle. Choice (E) is incorrect. To equivocate is to use language that is open to different interpretations and may mislead, confuse, or deceive. Denunciations are statements pronouncing something to be blameworthy or evil. The sentence

indicates that two different attitudes will be described. However, the difference between these attitudes will be subtle; this suggests that the two attitudes will actually be similar in some way. The difference between equivocating and making denunciations is not subtle: equivocations are confusing while denunciations are straightforward. 5 Explanation for Correct Answer D. Choice (D) is correct. Derivative means derived, taken, or obtained from something else. Inept means lacking in fitness or aptitude. The dancers moves were taken from other dancers, so it makes sense to describe his or her style as derivative. It also makes sense to describe the dancers style as inept if his or her moves were poorly executed, or not done well. Choice (A) is incorrect. Rousing means stirring, or giving rise to excitement. Memorable means notable or worth remembering. If the dancers moves were taken from other dancers and were poorly executed, or not done well, it is unlikely that his or her style would be described as either stirring or notable. Choice (B) is incorrect. Pedestrian means commonplace or unimaginative. Evolving means developing, or undergoing change. If the dancers moves were taken from other dancers and were poorly executed, or not done well, his or her style might be described as commonplace or unimaginative. However, nothing in the sentence suggests that the style is developing or changing in any way, so it is unlikely that it would be described as evolving. Choice (C) is incorrect. Chaotic means disordered, unpredictable, and confusing. Unprecedented means novel, new, or having no precedent. Although it is possible that poorly executed dance moves could be chaotic, or disordered, it is unlikely that the dancers style would be described as unprecedented, or new, if his or her moves were taken from other dancers. Choice (E) is incorrect. Spontaneous means arising from momentary impulse. Graceless means artistically inept or lacking beauty. Although the dancers style might certainly be described as graceless if his or her moves were poorly executed, or not done well, nothing in the sentence suggests that the dancers style is spontaneous. There is no connection between taking moves from other dancers and acting on momentary impulses. 6 Explanation for Correct Answer B. Choice (B) is correct. To transform something is to change it in character or condition. In this context, viable means having a reasonable chance of succeeding. The method developed by Lewis Latimer had some effect on the nascent electric industrythat is, the electric industry that had recently come into existence. It

makes sense to say that Latimer changed the condition of the electric industry when his inexpensive way of producing a necessary part made electric lamps commercially viable, or capable of succeeding in the commercial, or business, market. The electric industry surely benefited greatly from the ability to produce electric lamps inexpensively. Choice (A) is incorrect. In this context, cheapened means lowered in value. Affordable means inexpensive. The method developed by Lewis Latimer had some effect on the nascent electric industrythat is, the electric industry that had recently come into existence. Latimers inexpensive way of producing a necessary part might have led to an affordable, or inexpensive, price for electric lamps. However, it does not make sense to say that his method cheapened, or lowered the value of, the electric industry itself. This development likely raised the value of the industry and made those who worked in it very wealthy. Choice (C) is incorrect. To revolutionize something is to change it fundamentally or completely. Prohibitive means tending to rule out the use or purchase of something. The method developed by Lewis Latimer had some effect on the nascent electric industrythat is, the electric industry that had recently come into existence. Latimers method of producing a necessary part for electric lamps certainly might have revolutionized, or changed completely, the electric industry, but it does not make sense to suggest that this inexpensive method caused electric lamps to be commercially prohibitive. It is illogical to suggest that Latimers inexpensive method would rule out the use or purchase of the lamps. Choice (D) is incorrect. In this context, provoked means provided a necessary stimulus or boost. Improbable means unlikely to be true or to occur. The method developed by Lewis Latimer had some effect on the nascent electric industry that is, the electric industry that had recently come into existence. One might describe Latimers method of producing a necessary part for electric lamps as having provided a necessary stimulus if, for example, the method had spurred innovation in the electric industry. But it does not make sense to say that an inexpensive method of producing parts for electric lamps would make the lamps improbable. Choice (E) is incorrect. To stimulate something is to excite it to activity, growth, or greater activity. Inaccessible means not accessible, or unavailable for use. The method developed by Lewis Latimer had some effect on the nascent electric industrythat is, the electric industry that had recently come into existence. It is possible that Latimers method of producing a necessary part for electric lamps excited the electric industry to growth or further activity, but this idea is not made clear in the sentence. Further, it does not make sense to suggest that an inexpensive method of producing parts for electric lamps would make the lamps inaccessible, or unavailable for use. 7 Explanation for Correct Answer D.

Choice (D) is correct. Supercilious means coolly and patronizingly haughty. The sentence describes Phillips pose after he won the award as haughty, or blatantly and disdainfully proud, and indicates that he treated even his best friends in a similar manner. Therefore, it makes sense to say that Phillip treated his friends in a supercilious, or patronizingly haughty, manner. Choice (A) is incorrect. Cryptic means mysterious or seeming to have a hidden meaning. The sentence describes Phillips pose after he won the award as haughty, or blatantly and disdainfully proud, and indicates that he treated even his best friends in a similar manner. A cryptic, or mysterious, manner is not related to being blatantly proud, so the term cryptic is not the best choice. Choice (B) is incorrect. Judicious means having or being characterized by good judgment. The sentence describes Phillips pose after he won the award as haughty, or blatantly and disdainfully proud, and indicates that he treated even his best friends in a similar manner. Being judicious is not related to being blatantly proud. Additionally, one might say that behaving in a haughty way does not demonstrate good judgment. Choice (C) is incorrect. Jubilant means filled with or expressing great joy. Although Phillip might have been filled with joy after winning the award, the sentence describes his pose as haughty, or blatantly and disdainfully proud, and indicates that he treated even his best friends in a similar manner. Being joyful is not the same as being haughty, so the term jubilant is not the best choice. Choice (E) is incorrect. Pugnacious means having a combative nature, or being eager to fight. The sentence describes Phillips pose after he won the award as haughty, or blatantly and disdainfully proud, and indicates that he treated even his best friends in a similar manner. Nothing in the sentence suggests that Phillip was eager to fight, and being pugnacious is not the same as being blatantly proud. 8 Explanation for Correct Answer C. Choice (C) is correct. Perfidy means disloyalty. The generals name eventually became synonymous with disloyalty, so he must have been suspected of acts of disloyalty during the war. The term perfidy logically completes the sentence. Choice (A) is incorrect. Belligerence is an aggressive or harsh attitude. The generals name eventually became synonymous with disloyalty, so the suspicions about him during the war must have been related to disloyalty. An aggressive attitude is not necessarily related to being disloyal, so the term belligerence does not logically complete the sentence. Choice (B) is incorrect. Indigence is a level of poverty characterized by real hardship and deprivation. The generals name eventually became synonymous with disloyalty, so the suspicions about him during the war must have been related to

disloyalty. Being impoverished and suffering hardships are not related to being disloyal, so the term indigence does not logically complete the sentence. Choice (D) is incorrect. Aspersion is the act of making false or misleading charges in order to harm someones reputation. The generals name eventually became synonymous with disloyalty, so the suspicions about him during the war must have been related to disloyalty. Attempting to harm someones reputation could be seen as an act of disloyalty, but the sentence does not indicate that the general behaved in such a way, so the term aspersion is not the best choice. Rather, the general was suspected of perfidy, or disloyalty. Choice (E) is incorrect. Tenacity is the quality or state of being persistent. The generals name eventually became synonymous with disloyalty, so the suspicions about him during the war must have been related to disloyalty. Being persistent is not related to being disloyal, so the term tenacity does not logically complete the sentence. 9 Explanation for Correct Answer A. Choice (A) is correct. The author of Passage 2 and the teachers mentioned in Passage 1 would likely disagree about the extent to which Walden presents nature as being threatened. The author of Passage 1 states that teachers have used Walden as an example of the intensity with which nineteenth-century America protested the intrusion of industrialization and urbanization into pastoral harmonyin other words, many teachers have asserted that Walden presents nature as being threatened by industrialization. In contrast, the author of Passage 2 asserts that while Thoreau sometimes had mixed feelings about mechanization, or industrialism, at other times, he was downright enthusiastic about industrialism; the author also notes that "At Walden Pond, civilization and industrialization no longer seemed threatening." Choice (B) is incorrect. Although the author of Passage 1 refers to Walden as a text that recounts [Thoreaus] experiment in living in solitary harmony with nature, he or she does not discuss the teachers feelings about the texts success in recounting Thoreaus experiment. Nowhere in Passage 2 does the author address the idea that the text recounts Thoreaus experiment in solitary living. Therefore, we do not know if the author of Passage 2 and the teachers mentioned in Passage 1 would agree or disagree about how successfully Waldenrecounts Thoreaus experiment. Choice (C) is incorrect. The author of Passage 1 does describe Walden as being revered, or worthy of great honor, and one might assume that Generations of teachers have assigned it because it is an important work of literature. Nowhere in Passage 2, however, does the author discuss Walden as an important work of literature. Therefore, we do not know if the author of Passage 2 and the teachers mentioned in Passage 1 would agree or disagree about the extent to which Walden has been considered an important work of literature.

Choice (D) is incorrect. The author of Passage 1 states that teachers have used Walden as an example of the intensity with which nineteenth-century America protested the intrusion of industrialization and urbanization, so the teachers likely would say that Walden does recognize the spread of industrialization and urbanization. The author of Passage 2 notes that while Thoreau was at Walden Pond, civilization and industrialization no longer seemed threatening, so he or she would also likely say that Walden recognizes the spread of industrialization and urbanization. The author of Passage 2 and the teachers mentioned in Passage 1 would probably agree on this point, not disagree. Choice (E) is incorrect. The author of Passage 1 states that Generations of teachers have discussed Walden as generally protesting the intrusion of industrialization and urbanization, but he or she does not indicate whether the teachers consider the specific topic of the power of the machine to be central to Walden. Therefore, we do not know if the author of Passage 2 and the teachers mentioned in Passage 1 would agree or disagree regarding the power of the machine as a central topic in Walden. 10 Explanation for Correct Answer B. Choice (B) is correct. The author of Passage 1 describes Walden as a protest against the intrusion into pastoral harmony of the forces of industrialization and urbanization and states that Thoreau epitomizes a long-standing American worship of nature. Therefore, the author would likely say that Thoreau would agree that the power unleashed by the machine was largely destructive of natures tranquilityin other words, that industrialization and mechanization were harmful to nature. Choice (A) is incorrect. The author of Passage 1 describes Walden as a protest against the intrusion into pastoral harmony of the forces of industrialization and urbanization and states that Thoreau epitomizes a long-standing American worship of nature. He or she does not suggest that certain forces in nature might have kept the power unleashed by the machine in check, or that Thoreau would have agreed with that notion. Choice (C) is incorrect. One might view the quote from Waldenwritten by Thoreau, who did seek out naturein Passage 2 (shaking the earth with his feet, and breathing fire and smoke from his nostrils) as an exaggerated description of the power unleashed by the machine, but nothing in Passage 1 suggests that Thoreau would agree that this power is exaggerated by some people. Passage 1 does not address Thoreaus feelings about people who do not seek out nature. Choice (D) is incorrect. The author of Passage 2 suggests that Walden compares the power unleashed by the machine to the power required to transform the wilderness into a productive garden, but neither the author of Passage 2 nor the author of Passage 1 asserts that Thoreau thought mechanization was necessary to

transform nature into something productive. Further, the author of Passage 1 describes Walden as a protest against the intrusion into pastoral harmony of the forces of industrialization and urbanization; if Thoreau believed that the power unleashed by the machine was necessary to transform nature into something productive, it is unlikely that he would consider this power to be an intrusion. Choice (E) is incorrect. Passage 1 does not suggest that Thoreau would agree that the power unleashed by the machine was less threatening to one who lived close to nature. On the contrary, the author of Passage 1 states that Thoreau experimented with living in solitary harmony with nature, and that Walden, the product of this experiment, is a protest against the intrusion into pastoral harmony of the forces of industrialization and urbanization. Passage 1 suggests that Thoreau would argue that mechanization was threatening to nature and to those who lived close to nature. 11 Explanation for Correct Answer E. Choice (E) is correct. Passage 2 states that Thoreau was sometimes downright enthusiastic about mechanization and industrialization. The author of Passage 1, however, would likely argue that this enthusiasm is atypical of Thoreaus perceived attitude toward mechanization. People typically perceive Thoreau as being opposed to, not enthusiastic about, mechanization; Passage 1 states that Generations of teachers have used Walden as an example of nineteenth-century America's reaction against industrialization. Choice (A) is incorrect. Although the author of Passage 1 refers to Walden as a lament in response to the intrusion . . . of industrialization, the author of Passage 2 claims that, at times, Thoreau was downright enthusiastic about mechanization. Lament and enthusiasm are contradictory feelings; it is unlikely that one would argue that Thoreaus enthusiasm for machines supports the idea that Walden expresses regret about industrialization. Choice (B) is incorrect. Passage 2 states that Thoreau was sometimes downright enthusiastic about mechanization and industrialization. The author of Passage 1 states that nineteenth-century America protested the intrusion . . . of industrialization, so it is unlikely that he or she would argue that Thoreaus enthusiasm for certain machines would have resonatedthat is, related harmoniously or struck a chordwith nineteenth-century Americans. Choice (C) is incorrect. Passage 2 states that Thoreau was sometimes downright enthusiastic about mechanization and industrialization. The author of Passage 1 probably would not argue that this enthusiasm is a characteristic often emphasized by teachers; on the contrary, the author of Passage 1 says that Generations of teachers have used Walden as an example of a response against the intrusion . . . of industrialization.

Choice (D) is incorrect. Although Passage 2 states that in Walden Thoreau was sometimes downright enthusiastic about mechanization and industrialization, the author of Passage 1 probably would not argue that Thoreaus enthusiasm derived from his experiment in solitary living at Walden Pond. The author of Passage 1 says that Generations of teachers have usedWalden as an example of a response against the intrusion . . . of industrialization and that Thoreau worshipped nature, so he or she probably would not argue that Thoreaus experiment led to his enthusiasm for certain machines. 12 Explanation for Correct Answer C. Choice (C) is correct. The author of Passage 2 states that, while Thoreau was at Walden Pond, he found that civilization and industrialization no longer seemed threatening. The author also notes that at times Thoreau was downright enthusiastic about mechanization. The author of Passage 1 writes that Generations of teachers have drawn precisely the opposite conclusions about Walden, using the book as an illustration of the intensity with which nineteenth-century America protested the intrusion into pastoral harmony of the forces of industrialization and urbanization. The author of Passage 1 would surely agree that the interpretation of Walden in Passage 2 is not representative of the way the text is often taught in schools. Explanation for Incorrect Answer A. Choice (A) is incorrect. The author of Passage 1 suggests that Walden depicts machines as destructive: he or she writes that the text is a reaction against the intrusion into pastoral harmony of . . . industrialization. On the other hand, the author of Passage 2 focuses on the idea that Thoreau was at times downright enthusiastic about mechanization. The author of Passage 1 would likely argue that the interpretation offered by the author of Passage 2 underestimates, rather than exaggerates, the destructive power of the machine. Choice (B) is incorrect. The author of Passage 1 addresses the idea that Thoreau epitomizes a long-standing American worship of nature. The author of Passage 2 does not really address the worship of nature; on the contrary, he or she focuses on the positive aspects of mechanization, even suggesting that the power unleashed by the machine is similar to the power required to transform the wilderness into a productive garden. It is unlikely, then, that the author of Passage 1 would say that the interpretation offered in Passage 2 is overly influenced by a long-standing worship of nature. Choice (D) is incorrect. The author of Passage 2 explicitly discusses Thoreaus enthusiasm for the railroad. He or she notes Thoreaus reaction, as expressed in Walden: it seems as if the earth had got a race now worthy to inhabit it.

Because the author of Passage 2 directly addresses Thoreaus enthusiasm for the railroad, it is not likely that the author of Passage 1 would agree that he or she overlooks it. Choice (E) is incorrect. The author of Passage 1 does not discuss the way Walden was understood in Thoreaus time, so there is no reason to assume that he or she would agree that the interpretation of Walden in Passage 2 is more in accord with the way the text was understood in Thoreaus time than it is currently. 13 Explanation for Correct Answer D. Choice (D) is correct. A conception is a thought or an idea. In the first line, the author presents one vision of the city, that is, one idea about cities: that cities are unnatural. The author suggests that he or she is troubled by this particular idea or conception. Choice (A) is incorrect. A fantasy is a daydream or an unrealistic image. The idea that a city contains unnatural features is not merely a fantasy or a daydream. It is a well-established idea, and that bothers the author. Choice (B) is incorrect. An illusion is a false impression or a deception. Although the author believes that the people who hold one particular "vision" of the citythat cities are "unnatural"are wrong, the word "vision" does not mean illusion. In this context, the term vision" means an idea or conception. Choice (C) is incorrect. A prophecy is a prediction. The author does not use the word vision to indicate peoples predictions or forecasts of how cities will be in the future. Rather, the author refers to the well-established idea or conception that today's cities are unnatural. Choice (E) is incorrect. An apparition is a phantom or a spirit. When the author writes that he or she disagrees with one vision of the city, the author is referring to a real and well-defined notion that cities are "unnatural," not to a phantom or a spirit. In this context, the term "vision" means an idea or conception. 14 Explanation for Correct Answer D. Choice (D) is correct. Throughout the passage, the author disputes the claim that cities are "unnatural." The second paragraph presents the argument used in support of that claim ("The argument goes like this"). Given that the author disagrees with the claim, it follows that the author would disagree with the supporting argument as well. One of the points made in the argument is that there was a "happier state of existence" before the growth of cities. It is likely that the author would describe that so-called "happier state" as a false supposition, or an inaccurate idea.

Choice (A) is incorrect. The argument that abandoning cities and technologies and returning to nature will result in a happier state of existence is a notion the author rejects (one vision of the city really gets my hackles up). Therefore, the author would most likely not describe this return to nature as a satisfactory solution to problems associated with the city. Choice (B) is incorrect. Throughout the passage, the author disputes the claim that cities are "unnatural." The second paragraph presents the argument used in support of that claim ("The argument goes like this"). Given that the author disagrees with the claim, it follows that the author would disagree with the supporting argument as well. One of the points made in the argument is that there was a "happier state of existence" before the growth of cities. Therefore, the author would most likely not describe the happier state as an agreeable stroke of luck. Choice (C) is incorrect. Throughout the passage, the author strongly disagrees with the claim that cities are "unnatural" and with the supporting argument. In the second paragraph the author describes the argument, which claims that we should abandon cities and return to a happier state of existence. There is no indication that the author would see this condition as a complicated arrangement. Choice (E) is incorrect. The author does not believe that to be part of nature one needs to return to an earlier or happier state of existence. Rather than considering this happier state to be a bittersweet memory, the author states that the whole idea bothers him or her and puts forward an alternative vision of the city as a natural system. 15 Explanation for Correct Answer E. Choice (E) is correct. When the author mentions people who think this way, he or she refers to those who support the argument that nature, left to itself, will find a state of equilibrium and that humanity should find a way to adjust to that balance. The author suggests that following this line of argument leads one to believe that the Industrial Revolution, which is associated with modern technology and the growth of cities, "represents a wrong turning." In other words, people who think this way view the Industrial Revolution as the beginning of a harmful trend in human history. Choice (A) is incorrect. The Industrial Revolution may be considered an important human achievement in the history of humankind. However, the authors reference in line 18 to people who think this way points specifically to those who think that the Industrial Revolution brought about a negative resultthat is, the destruction of nature. Choice (B) is incorrect. It may well be that the Industrial Revolution is an instance of technologys double-edged potential, or of both positive and negative developments. However, the authors mention of people who think this way refers to those who believe the Industrial Revolution had only a negative outcomethat

the Industrial Revolution caused much harm to the natural environment and destroyed the balance between nature and humanity. Choice (C) is incorrect. When the author refers to people who think this way, he or she does not refer to those who believe that the Industrial Revolution made cities self-sufficient and balanced. Rather, the author refers to people who think that the Industrial Revolution destroyed the balance between nature and humanity. Choice (D) is incorrect. The Industrial Revolution may well have happened during a time when people were beginning to go beyond strict social distinctions. However, when the author refers to people who think this way, he or she does not refer to this particular social transformation. Rather, the author refers to people who believe that the Industrial Revolution brought about much harm to the environment and helped destroy the balance of nature. 16 Explanation for Correct Answer E. Choice (E) is correct. The environmental thinkers referred to in line 28 believe that large populations and cities contribute to the destruction of nature. The author does not agree with this line of argument (What bothers me . . . is that it implies that human beings . . . are not part of nature). Therefore, the author would most likely characterize the views of the environmental thinkers as erroneous, or wrong. Choice (A) is incorrect. According to the author, the argument made by the environmental thinkers has a flaw: it implies that human beings . . . are not part of nature. Therefore, the author probably would not suggest that their argument is carefully reasoned; rather, he or she points out that their argument does not take into consideration the possibility that cities are part of nature. Choice (B) is incorrect. While the argument put forth by the environmental thinkers may be thought-provoking for some, the author probably would not characterize it as such. Instead, the author points out that the argument bothers him or her (What bothers me about this point of view is that it implies that human beings . . . are not part of nature). Choice (C) is incorrect. Unintelligible refers to something that is not comprehensible. The author understands the argument put forth by the environmental thinkers well enough to point out the flaw in the thinkers reasoning (it implies that human beings . . . are not part of nature), so it is not unintelligible. Choice (D) is incorrect. According to the author, the argument made by the environmental thinkers is conclusive; that is, it concludes that cities are not part of nature. The author disagrees with that conclusion and proposes another line of argument, stating that cities are natural systems.

17 Explanation for Correct Answer B. Choice (B) is correct. The author proposes that cities are the product of humanitys natural inclination to survive and to build shelters. By comparing cities to beaver dams and anthills, the author is suggesting that all three are part of a survival strategyor products of natural impulsesby which humans, beavers, and ants alter their environments and build shelters in order to survive. Choice (A) is incorrect. The purpose of the authors comparison of cities to beaver dams and anthills is not to explain the workings of ecological systems. Rather, it is to suggest that all three products are the result of a "survival strategy" by which humans, like beavers and ants, engage in altering their environments in order to survive. Choice (C) is incorrect. The author does not suggest that cities, beaver dams, and anthills are detrimental to nature. On the contrary, the author asserts that they are products of naturethat is, they represent the natural impulses of humans, beavers, and ants to survive by altering their environments and creating shelters. Choice (D) is incorrect. The author does not compare cities, beaver dams, and anthills in order to point out that different forms of life are products of different environments. Rather, the author compares these three natural environments to emphasize something similar in all of them: they are all products of a natural inclination to survive. Choice (E) is incorrect. The author does not compare cities, beaver dams, and anthills in order to show obstacles facing cities today. Rather, the comparison calls attention to the positive function of a city. Humans use their natural instincts to create cities for the purpose of continuing their existence, just as beavers and ants alter their environments to create shelters for their survival. 18 Explanation for Correct Answer E. Choice (E) is correct. To appreciate something is to understand and admire it. The author demonstrates that he or she understands that downtown areas are, in fact, natural, and admires them for it, by offering several examples of the citys natural qualities: Look at the energy sources of the downtown ecosystem . . ." The fact that the author even values the city's trash ("hamburger buns, apple cores, and partially filled soft-drink containers"), as a food source, shows that the author's attitude toward the downtown ecosystem is positiveit is an attitude of appreciation. Choice (A) is incorrect. To have "regret" is to feel sorry or apologetic. When the author describes a downtown ecosystem, there is no hint of regret or apology. On

the contrary, the author demonstrates a sense of pride in the city by pointing out that there are energy sources for providing warmth, food sources that feed animals, and sugar-rich soft drink cans in waste containers that provide an excellent source of nectar for yellow jackets. Choice (B) is incorrect. In describing how a downtown area is an ecosystem, the author does not indicate that he or she feels frustrated or irritated. Instead, the author admiringly calls attention to the interdependence of the city and its inhabitants by asking the reader to consider all the different ways downtown areas provide food (there is a large amount of human-made detritus that can serve as food for animals. . . . urban yellow jackets seem to find sugar-rich soft drink cans an excellent source of nectar for their honey). Choice (C) is incorrect. The author does not display any "ambivalence," or uncertainty, toward the downtown ecosystem. On the contrary, the author gives numerous examples of how nothing is wasted in downtown areas to demonstrate his or her firm conviction that cities are natural systems. Choice (D) is incorrect. When the author describes the downtown ecosystem, he or she is not unconcerned, or indifferent. The author seems to admire how waste in downtown areas can become a source of food for animals and insects. The author demonstrates his or her enthusiasm for downtown ecosystems by offering several examples and by imploring readers to take note of how efficient these ecosystems are. 19 Explanation for Correct Answer A. Choice (A) is correct. The discussion in lines 57-81 of "three different levels" presents a number of related arguments in support of the idea that the city is a natural system." The first sentence of the paragraph beginning on line 57 states clearly that "the city can be thought of as a natural system on at least three different levels." The lines that follow explain the three levels; in each level the author presents, with reasoning and evidence, ways in which the city is like an ecosystem. Choice (B) is incorrect. Nothing in lines 57-81 indicates that the author has organized his or her opinions about the city from most to least important. Instead, the author organizes his or her arguments according to their level of complexity: from the most obvious illustration of the city as an ecosystem to the more complex or deeper explanations. The author's overall point is that the fundamental structures of cities can be seen as being the same as those of natural systems. Choice (C) is incorrect. Lines 57-81 do not present a process of reasoning by which the author makes an initial assertion and ends with an ultimate conclusion. In these lines, the author supports and strengthenswith three different, but related, exampleshis or her main argument that the city is a natural system.

Choice (D) is incorrect. In lines 57-81 the author expresses one main hypothesis and backs it up with three different examples: "the city can be thought of as a natural system on at least three different levels." There is no evidence of opposing principles. The single hypothesis that the author supports is that the city is a natural system. Choice (E) is incorrect. Lines 57-81 do not express alternative or competing theories. Rather, they express the authors main theory that the city is a natural system that can be observed and studied like other natural systems: "the city can be thought of as a natural system on at least three different levels." 20 Explanation for Correct Answer D. Choice (D) is correct. In lines 61-63, the author suggests that all ecosystems have their own combinations of animal and plant life and states that the city has its own peculiar combination. In this context, the term peculiar means distinctive, or characteristic of only one person, group, or thing. The author makes the point that like any other ecosystem, the city has a distinctive blend of animal and plant life. Choice (A) is incorrect. In lines 61-63, the author suggests that all ecosystems have their own combinations of animal and plant life and states that the city has its own peculiar combination. One might find the combination of animals and plants in the city to be eccentric, or strange, but the author does not use the word peculiar to suggest this. Rather, the author suggests that like any other ecosystem, the city has a distinctive, or unique, collection of flora and fauna. Choice (B) is incorrect. In lines 61-63, the author suggests that all ecosystems have their own combinations of animal and plant life and states that the city has its own peculiar combination. The author does not use the word peculiar to imply that the combination of animals and plants in the city is abnormal, or that it deviates from what is normal. Rather, the author suggests that it is normal for every ecosystem to have its own collection of flora and fauna; like any other ecosystem, the city has a distinctive, or unique, blend of animal and plant life. Choice (C) is incorrect. In lines 61-63, the author suggests that all ecosystems have their own combinations of animal and plant life and states that the city has its own peculiar combination. The author makes the point that like any other ecosystem, the city has a distinctive, or unique, collection of flora and fauna. The author does not imply that the animals and plants found in the city are rare, or not commonly found. Choice (E) is incorrect. In lines 61-63, the author suggests that all ecosystems have their own combinations of animal and plant life and states that the city has its own peculiar combination. The author might agree that the collection of flora and fauna found in the city is significant, or noteworthy, but he or she does not use

the word peculiar to suggest this. Rather, the author suggests that like any other ecosystem, the city has a distinctive, or unique, blend of animal and plant life. 21 Explanation for Correct Answer A. Choice (A) is correct. The author clearly approves of the creation of urban ecologyhe or she calls the creation of the subfield the ultimate academic accolade, or a great mark of acknowledgment. The subfield, which is designed to study cities as ecosystems, gives legitimacy to the authors stance that cities are natural systems. Choice (B) is incorrect. Curiosity is not the best characterization of the authors attitude toward the creation of the subfield of urban ecology. The author writes from a knowledgeable point of view; he or she seems to be an authority on urban ecology. Rather than being curious, the author seems pleased that this way of looking at cities" is receiving the recognition and scientific legitimacy it deserves. Choice (C) is incorrect. There is no indication that the author is uncertain about the creation of the subfield of urban ecology. The author supports this new way of looking at cities and is thus very pleased with this new subfield. In fact, he or she describes the creation of the subfield as the ultimate academic accolade, or a great mark of acknowledgment. Choice (D) is incorrect. Given that the author is a strong supporter of the notion that the city is a natural system, it should not come as a surprise to him or her that a subfield of science designed to study urban ecology has been created. Rather, the author sees the creation of this new subfield as an opportunity for science to further its understanding of the city as a natural system. Choice (E) is incorrect. The author is not dismayed, or disappointed, by the creation of the subfield of urban ecology. On the contrary, the author is a strong supporter of the point of view that the city is a natural system, and he or she considers the creation of the subfield to be a positive development (the ultimate academic accolade). 22 Explanation for Correct Answer C. Choice (C) is correct. The discussion in lines 67-73 is best characterized as a comparison. In these lines, the author lays out the similarities between the evolution of a forest and the transformation of a city in order to illustrate how the city is a functioning ecosystem. The use of the word "metaphor" ("a forest is a powerful metaphor to aid in understanding how cities work") suggests that a city and a forest have much in common. The remainder of the paragraph compares the two, focusing on their similarities.

Choice (A) is incorrect. In lines 67-73, the author does not defend his or her position in response to any particular argument. Instead, the author supports the claim that a city is an ecosystem by presenting a metaphor to aid in understanding how cities work: the author compares the city and a forest ecosystem, focusing on their similarities. Choice (B) is incorrect. The authors discussion of a forest ecosystem cannot be characterized as a concession, or an acknowledgment that part of an opposing argument is correct. The author does not back away from his or her argument that a city is a natural system; rather, the author compares the city and a forest ecosystem, focusing on their similarities, with the goal of supporting his or her claim. Choice (D) is incorrect. The discussion of a forest ecosystem cannot be characterized as an exception, or unusual case. Rather, the comparison of a forest and a city supports the main point of the passagethat a city functions in the same way as a natural system. Choice (E) is incorrect. An allusion is a mention or a passing reference. In lines 67-73, the author demonstrates that a city and a forest ecosystem share similar fundamental structures. This comparison is not just an allusion or a passing reference. It is an important comparison that supports the authors argument. 23 Explanation for Correct Answer E. Choice (E) is correct. In lines 74-81, the author discusses limits in order to point out that similar laws of nature guide all natural systems, including those shaped by humans. Just as the height of a tree is governed by the various forces that exist between atoms in wood, so is the height of a buildingwhether it is made of wood or another materialinfluenced by those same interatomic forces. Choice (A) is incorrect. While it may be true that cities have the ability to change and grow, the main purpose of the discussion of limits is not to show how cities change and grow. Rather, the author discusses limits in order to point out that similar laws of nature guide all natural systems, including those shaped by humans. Choice (B) is incorrect. Nothing in lines 74-81 suggests that the author believes cities are too large. The authors point in these lines is that cities, like other natural systems, are governed by certain laws of nature. Choice (C) is incorrect. Nothing in lines 74-81 suggests that the author thinks a city must be self-regulating. The author discusses limits in order to point out that similar laws of nature guide all natural systems, including those shaped by humans. It is clear that cities are regulated in part by natural laws. Choice (D) is incorrect. The discussion of limits does not suggest that efforts to conquer nature backfire, or fail. The author simply acknowledges that human efforts

to shape nature are subject to natural lawsbuildings cannot be built infinitely tall, for exampleas a way of showing that cities "are like every other natural system." 24 Explanation for Correct Answer E. Choice (E) is correct. In the last paragraph, the author emphasizes, or stresses, the point he or she makes throughout the entire passage: A city is a natural system and we should be able to observe it in the same way we observe other natural systems. Choice (A) is incorrect. In the last paragraph, the author does not restate the elements of a dilemma. To the author, there is no dilemma, or difficult choice. Instead, he or she clearly restates the position he or she takes throughout the passage: the city should be considered a natural system. Choice (B) is incorrect. In the last paragraph, the author does not summarize evidence. Instead, the author emphasizes his or her belief that cities are natural systems. The author discusses evidence for this claim throughout the passage. Choice (C) is incorrect. In the last paragraph, the author does not attempt to heighten or intensify the emotional impact of his or her argumentthe author's argument appeals to logic, not emotion. Rather, the final paragraph serves to emphasize, or stress, the point the author makes throughout the passage: the city should be considered a natural system. Choice (D) is incorrect. In the last paragraph, the author does not suggest further research. Rather, the author uses the final paragraph to emphasize, or stress, the point that he or she makes throughout the entire passage: the city should be considered a natural system.

Section #6: View Explanations 1 Explanation for Correct Answer C. Choice (C) is correct. It avoids the comma-splice error of the original by replacing the main verb fall with a phrase (by falling...) that modifies the verb caused. Choice (A) involves a comma splice. Two independent clauses (A recent report indicates . . . year and they fall asleep at the wheel) are joined by only a comma. Choice (B) results in improper subordination. The first clause (sleep-deprived drivers caused more than 100,000 accidents last year) is the result of the second

clause (they fall asleep at the wheel). The conjunction and does not adequately show this cause-and-effect relationship. Choice (D) results in improper coordination. The coordinating conjunction and should be replaced with the preposition by to indicate a cause-and-effect relationship. Choice (E) results in a comma splice. Two independent clauses (A recent report indicates...year and they were falling asleep at the wheel) are joined by only a comma. 2 Explanation for Correct Answer C. Choice (C) is correct. It avoids the awkwardness and redundancy of the original by eliminating the unnecessary verb being. Choice (A) involves awkwardness and redundancy. The verb form being is awkward and unnecessary and should be deleted. Choice (B) results in improper subordination. It is not clear what the adverb mainly is meant to modify. The relative pronoun which is needed instead, to indicate why the depths of the Arctic Ocean are hard to study (... as a result of the icy surface, which is difficult to penetrate using current techniques). Choice (D) results in awkward, unidiomatic phrasing. The phrase are hard studying is awkward and unidiomatic. The sentence requires the infinitive form of the verb, to study, instead. In addition, the word from is not idiomatic in this context. The word because should be used instead, to indicate a cause-effect relationship (because the icy surface is difficult to penetrate). Choice (E) results in an error in verb form. The sentence requires the infinitive form of the verb, to study. In addition, the gerund being requires that the noun surface be in the possessive form (the icy surfaces being difficult to penetrate). 3 Explanation for Correct Answer B. Choice (B) is correct. It avoids the sentence-fragment error of the original by providing a past-tense plural verb phrase (were caused) as the action of the subject fires. Choice (A) involves a sentence fragment. There is no main verb to carry out the action of the subject fires, only the subordinate clause that occurred last summer and the awkward modifying phrase which were because people are careless.

Choice (C) results in a sentence fragment. There is no main verb to carry out the action of the subject fires, only the subordinate clause that occurred last summer and the dependent clause that begins with because. Choice (D) results in a verb-tense error. The phrase last summer indicates an action that occurred in the past, so the present-tense verb are is inappropriate. Choice (E) results in vague, awkward phrasing. The verb phrase happened from does not clearly indicate a cause-and-effect relationship between forest fires and peoples carelessness. In addition, the phrase that occurred last summer followed by happened is repetitive. 4 Explanation for Correct Answer C. Choice (C) is correct. It avoids the verb-tense error of the original by using the simple past-tense verb disproved (as indicated by the past-tense verb showed). Choice (A) involves inappropriate verb tense. The past perfect tense (has disproved) is used to indicate an action that is ongoing; however, the clause that begins with when indicates an action completed in the past (when she showed...). The past-tense verb disproved should be used instead. Choice (B) results in a sentence fragment. There is no main verb to carry out the action of the sentence, only the participial phrase having disproved. Choice (D) involves inconsistent verb tense that results in an illogical statement. The action of the sentence takes place in the past (when she showed . . .), so the main verb in the sentence should be in the past tense (disproved). Choice (E) results in a sentence fragment. There is no main verb to carry out the action of the sentence, only the participle disproving. 5 Explanation for Correct Answer E. Choice (E) is correct. It avoids the coordination error of the other options by providing an appropriate conjunction (however) to indicate a contrast between the first and second independent clauses. Choice (A) involves improper coordination. There is a contrast between the clause We generally think of Canada as the northern neighbor of the United States and the clause more than half of the states extend farther north... The conjunction and does not establish this contrast and should be replaced with a conjunction such as however, which shows contrast and correctly joins the two independent clauses.

Choice (B) results in improper coordination and wordiness. The conjunction and does not adequately establish contrast between the first and second clauses. In addition, it is the case that is unnecessary and should be deleted. Choice (C) results in faulty sentence structure. There is no main verb to carry out the action of the second independent clause, only the participle extending. Choice (D) results in an illogical phrase. It does not make sense to say that the states are extending as if the extending is a continuing action or the states are in the process of extending. It is more appropriate to say that the states extend. 6 Explanation for Correct Answer A. Choice (A) is correct. It avoids the errors of the other options by providing the present-tense plural verbs begin and culminate, which are actions completed by the plural subject (volumes). Choice (B) results in a sentence fragment. There is no main verb to carry out the action of the sentence, only the relative clause that begins with that. Choice (C) results in a lack of parallelism and an error in sentence structure. The two verb forms (have begun and culminating) are not parallel, and the participle culminating cannot serve as a main verb. Choice (D) results in a sentence fragment. The participles beginning and culminating modify the subject volumes, but neither is a main verb that can serve to carry out the action of the sentence. Choice (E) involves unidiomatic, confusing phrasing. The verb phrase are begun is not idiomatic. In addition, it does not make sense to say that the memoirs are begun as a child. 7 Explanation for Correct Answer D. Choice (D) is correct. It avoids the modification error of the original by changing the modifying phrase (Dressed in a crisp, clean uniform), which cannot logically modify the subject it, to the subject of the sentence (The crisp, clean uniform). Choice (A) involves illogical modification. It does not make sense to say that it would be Dressed in a crisp, clean uniform. Choice (B) results in illogical modification. It does not make sense to say that the efficient manner of the tour guide was Dressed in a crisp, clean uniform.

Choice (C) results in a sentence fragment. There is no subject to carry out the action of the sentence. Choice (E) results in a sentence fragment. There is no main verb to carry out the action of the sentence. 8 Explanation for Correct Answer A. Choice (A) is correct. It avoids the errors of the other options by providing logical modification, specific language, and a main verb to make a complete sentence. Choice (B) results in illogical pronoun reference. There is nothing in this sentence to which the pronoun they can logically refer. Choice (C) results in a sentence fragment. There is no main verb to carry out the action of the sentence, only the participle being. Choice (D) results in illogical modification. It does not make sense to describe scientists as A cure for some kinds of cancer. Choice (E) results in a sentence fragment. There is no main verb to carry out the action of the sentence, only the modifying phrase which, some scientists believe, may be found... 9 Explanation for Correct Answer E. Choice (E) is correct. It avoids the errors of the original by providing a phrase (elegant because of its melodious final movement) that is parallel with the phrase that precedes it (confusing because of its unusual structure) and is coordinated with the phrase called it, and a conjunction (but) that logically contrasts the two opposing ideas. Choice (A) involves errors in parallelism and coordination. In this sentence, the phrase called it requires parallel phrases joined by a logical conjunction. The phrase its melodious final movement makes it elegant is not parallel with confusing because of its unusual structure and does not coordinate with the phrase called it (called it . . . its melodious final movement makes it). Choice (B) results in a lack of parallelism. The phrase by having its melodious final movement is not parallel with the phrase that precedes it (because of its unusual structure) and results in an awkward sentence. Choice (C) results in errors in parallelism and coordination. In this sentence, the phrase called it requires parallel phrases joined by a logical conjunction. The phrase it is elegant with its melodious final movement is not parallel with

confusing because of its unusual structure and does not coordinate with the phrase called it (called it . . . it is elegant). Choice (D) results in errors in parallelism and coordination. In this sentence, the phrase called it requires parallel phrases joined by a logical conjunction. The phrase while having a melodious final movement is not parallel with confusing because of its unusual structure and does not coordinate with the phrase called it (called it . . . while having). 10 Explanation for Correct Answer C. Choice (C) is correct. It avoids the errors of the original by replacing the modifying phrase "By building new windmill farms, which cannot logically modify the subject of the sentence ("consumption and "tons), with the gerund "Building," which functions as the singular subject of the sentence and agrees with the singular verbs "reduces and "keeps. Choice (A) involves improper modification, which results in confusing phrasing, and subject-verb disagreement. Because the modifying phrase "By building new windmill farms does not contain a subject, it is not clear who or what is doing the building. In addition, the singular subject "consumption does not agree with the plural verb phrase "are reduced. Choice (B) results in confusing phrasing and vague pronoun reference. Because the modifying phrase "By building new windmill farms does not contain a subject, it is not clear who or what is doing the building. In addition, there is no clear antecedent to which "it ("it reduces) can refer. Choice (D) results in a pronoun shift. The pronoun "they ("they reduce) refers to windmills; however, there is no clear antecedent to which "it ("it keeps) can refer. Choice (E) results in lack of parallelism. The active verb phrase "reduce fossil fuel is not parallel with the passive verb phrase "emissions are kept. 11 Explanation for Correct Answer E. Choice (E) is correct. It avoids the coordination error of the original by clearly setting the phrase in the Scottish Highlands in opposition to the first part of the sentence with the adverb "not." Choice (A) involves improper coordination and results in an illogical statement. The coordinating conjunction but is needed in place of and to indicate a contrast between the two independent clauses. In addition, it does not make sense to say that people assume the battle (the only noun to which the pronoun it might refer) was the Scottish Highlands.

Choice (B) results in improper subordination. The coordinating conjunction but is needed to indicate a contrast between the two ideas in the sentence (but many people...). In addition, the participle assuming should be replaced with the present-tense assume (but many people assume it was in the Scottish Highlands). Choice (C) results in an illogical statement. It does not make sense to say that people assume the battle (the only noun to which the pronoun it might refer) to be the Scottish Highlands. Choice (D) results in awkward and confusing phrasing. There is nothing in the sentence to which what can refer. In addition, in this context it is not idiomatic for assume to be followed by only a noun rather than an entire clause. 12 Explanation for Correct Answer C. The error in this sentence occurs at (C). The plural subject Larissa and Tariq does not agree with the complement an entomologist. The singular noun should be replaced with the plural entomologists. There is no error at (A). The preposition in appropriately introduces the prepositional phrase in studying insects and their effects, and the gerund studying serves as the object of the preposition. There is no error at (B). The infinitive phrase to become appropriately functions as the direct object of the verb plan. There is no error at (D). The plural verb return agrees with the plural subject Larissa and Tariq, and the infinitive phrase to help appropriately functions as the direct object of the verb return. There is an error in the sentence. 13 Explanation for Correct Answer E. The correct choice is (E). This sentence contains no errors. There is no error at (A). The preposition From appropriately introduces the prepositional phrase From about A.D. 700 to 1600, which modifies the verb created. There is no error at (B). The past tense of the verb created properly indicates an event that happened in the past (From about A.D. 700 to 1600).

There is no error at (C). The adverb nearly appropriately modifies the adjective 1,000. There is no error at (D). The adjective remote appropriately modifies the proper noun Easter Island. 14 Explanation for Correct Answer A. The error in this sentence occurs at (A). In the context of this sentence, the casserole was smelling logically means that the casserole was emitting odor, not perceiving odor. To indicate this meaning, the adverb badly should be replaced with the adjective bad. The adverb surprisingly would then appropriately modify the adjective bad. There is no error at (B). The pronoun us appropriately functions as the indirect object of the verb to ask, and the pronoun what functions as the direct object. There is no error at (C). The past perfect tense of the verb had put is appropriate to indicate an action that was completed before a certain time in the past. There is no error at (D). The prepositional phrase in it appropriately modifies the verb had put. The pronoun it refers to the noun casserole. There is an error in the sentence. 15 Explanation for Correct Answer C. The error in this sentence occurs at (C). The actions of the subject occurred in the past, as indicated by the past-tense verbs referred and learned, so the presenttense verb seeks out should be replaced with the past-tense verb sought out. There is no error at (A). The preposition to idiomatically follows the verb referred and appropriately introduces the prepositional phrase to art history textbooks. There is no error at (B). The conjunction while serves to link the action of referred to art history textbooks with the action of was sculpting. The pronoun he properly refers to the subject Jerome. There is no error at (D). The past perfect verb had used appropriately indicates an action that was completed before a certain time in the past. The pronoun it is the direct object of had used and appropriately refers to the noun method. There is an error in the sentence.

16 Explanation for Correct Answer D. The error in this sentence occurs at (D). The form of the verb suppressing is inappropriate and should be replaced with the infinitive form to suppress. There is no error at (A). The adverb eagerly appropriately modifies the verb awaited. There is no error at (B). The preposition for appropriately introduces the prepositional phrase for the job, which modifies interview. There is no error at (C). The verb thought appropriately expresses an action completed by the subject in the past, and the indefinite pronoun it functions as the direct object of the verb thought. The adjective wise appropriately modifies the pronoun it. There is an error in the sentence. 17 Explanation for Correct Answer C. The error in this sentence occurs at (C). An adverb is required to modify the verb phrase has risen, so the adjective noticeable should be replaced with the adverb noticeably." There is no error at (A). The complex preposition According to appropriately introduces the prepositional phrase According to educational statistics. There is no error at (B). The present perfect tense of the verb phrase has risen appropriately indicates an action that is completed in the present, and this singular verb phrase agrees with the singular subject age. There is no error at (D). The preposition over appropriately introduces the prepositional phrase over the past 25 years, which modifies the verb phrase has risen. There is an error in the sentence. 18 Explanation for Correct Answer A. The error in this sentence occurs at (A). It is not idiomatic to follow the correlative conjunction Neither with or, which should be replaced with nor.

There is no error at (B). The singular verb belongs agrees with the singular subject panda. In addition, the preposition to appropriately introduces the prepositional phrase to the bear family, which modifies the verb belongs. There is no error at (C). The singular verb is agrees with the singular subject panda. There is no error at (D). The complement related describes the subject panda, and the preposition to idiomatically follows related and appropriately introduces the prepositional phrase to the raccoon. There is an error in the sentence. 19 Explanation for Correct Answer C. The error in this sentence occurs at (C). The singular possessive pronoun phrase his or her refers to the plural noun passengers and should be replaced with the plural possessive pronoun their. There is no error at (A). The gerund boarding appropriately functions as the object of the preposition Before. There is no error at (B). The auxiliary verb must appropriately indicates that the verb purchase is imperative. There is no error at (D). The conjunction because appropriately functions as the link between the effect passengers must purchase . . . tickets in the main concourse and the cause tickets are not sold on the bus. There is an error in the sentence. 20 Explanation for Correct Answer E. The correct choice is (E). This sentence contains no errors. There is no error at (A). The complex preposition According to appropriately introduces the prepositional phrase According to some demographers, which modifies the verb is. There is no error at (B). The conjunction or appropriately links the adjective 65 and the adjective older. There is no error at (C). The singular verb is agrees with the singular subject number.

There is no error at (D). The infinitive of the verb to rise appropriately serves as the direct object of the verb is. 21 Explanation for Correct Answer B. The error in this sentence occurs at (B). There is an illogical comparison between Norman Rockwells paintings and painter Robert Rauschenberg (paintings to person). To create a logical comparison, to painter Robert Rauschenberg should be replaced with to Robert Rauschenbergs paintings. There is no error at (A). The plural past-tense verb phrase were asked agrees with the plural subject they and appropriately indicates an action completed in the past. There is no error at (C). The past-tense verb entered appropriately indicates an action by the students that takes place in the past. The preposition into idiomatically follows the verb entered and appropriately introduces the prepositional phrase into a prolonged discussion. There is no error at (D). The adjective prolonged appropriately modifies the noun discussion. There is an error in the sentence. 22 Explanation for Correct Answer E. The correct choice is (E). This sentence contains no errors. There is no error at (A). The complex preposition Contrary to appropriately introduces the prepositional phrase Contrary to what many people believe, and the pronoun what serves as the object of the preposition. There is no error at (B). The singular verb is agrees with the singular subject lightning. The adverb not appropriately serves to make negative the second occurrence of the noun lightning. There is no error at (C). The prepositional phrase at too great a distance appropriately modifies the verb occurs. There is no error at (D). The adjective audible appropriately functions as the direct object of the infinitive verb to be. 23 Explanation for Correct Answer C.

The error in this sentence occurs at (C). The singular verb phrase is released does not agree with the plural subject spears and should be replaced with the plural verb phrase are released. There is no error at (A). The singular pronoun each appropriately refers to a single spear that is trailed by a poison thread. There is no error at (B). The past participle trailed appropriately modifies the pronoun each, and the preposition by idiomatically follows the part participle trailed and introduces the prepositional phrase by a poison thread. There is no error at (D). The adverb when appropriately modifies the verb released. There is an error in the sentence. 24 Explanation for Correct Answer E. The correct choice is (E). This sentence contains no errors. There is no error at (A). The complex preposition across from appropriately introduces the prepositional phrase across from the warehouses, which modifies the noun house. There is no error at (B). The adverb more appropriately modifies the adjective decrepit, and the adverb even modifies more. There is no error at (C). The adverb just appropriately modifies the implied verb is (the . . . one [that is] just beside them). The preposition beside introduces the prepositional phrase beside them, which modifies the pronoun one. There is no error at (D). The present perfect tense have been placed appropriately indicates an action that is completed in the present (i.e., the houses are on the list now, in the present). 25 Explanation for Correct Answer C. The error in this sentence occurs at (C). The preposition to does not idiomatically follow the adjective capable and should be replaced with of. The form of the verb distinguish should then be replaced with distinguishing to appropriately follow the preposition of. There is no error at (A). The conjunction as appropriately functions as a link between the concept their [infants] brains mature and the concept infants become more capable.

There is no error at (B). The plural verb mature agrees with the plural subject brains, and the adverb neurologically appropriately modifies the verb mature. There is no error at (D). The prepositional phrase around them appropriately modifies the noun objects. There is an error in the sentence. 26 Explanation for Correct Answer D. The error in this sentence occurs at (D). The plural possessive pronoun their does not agree with the singular noun Hershey, Pennsylvania and should be replaced with the singular possessive pronoun its. There is no error at (A). The noun Home functions as an appositive to rename the proper noun Hershey, Pennsylvania. The preposition of appropriately introduces the prepositional phrase of the worlds largest chocolate-manufacturing plant. There is no error at (B). The adverb originally appropriately modifies the adjective known. The preposition as idiomatically follows the adjective known and introduces the prepositional phrase as Derry Church. There is no error at (C). The preposition to appropriately serves to introduce the infinitive form to honor, which modifies the verb phrase was changed. There is an error in the sentence. 27 Explanation for Correct Answer E. The correct choice is (E). This sentence contains no errors. There is no error at (A). The preposition in appropriately introduces the prepositional phrase in this display, which modifies the noun instruments. There is no error at (B). The plural pronoun all appropriately refers to the plural noun instruments. The adverb more, linked by the conjunction than, modifies the adjective 300. There is no error at (C). The preposition by appropriately introduces the prepositional phrase by artisans famous, which modifies the verb crafted. There is no error at (D). The adverbs long and since appropriately modify the adjective forgotten.

28 Explanation for Correct Answer D. The error in this sentence occurs at (D). The singular verb requires does not agree with the plural subject grades and should be replaced with the singular verb require. There is no error at (A). The adjective scheduled appropriately modifies the noun conference, and the adverb regularly modifies the adjective scheduled. There is no error at (B). The objective case of the pronoun is appropriate because my tutor and me are the objects of the preposition between. There is no error at (C). The singular verb is agrees with the singular subject conference. There is an error in the sentence. 29 Explanation for Correct Answer D. The error in this sentence occurs at (D). The sentence illogically compares story to Hank Aaron. To create a logical comparison, the phrase that of should be inserted after than so that the sentence logically compares no story to that [the story] of . . . Hank Aaron. There is no error at (A). The pronoun There appropriately introduces the sentence. The singular verb is agrees with the singular subject story. There is no error at (B). The adverb probably appropriately modifies the verb is, and the adjective no modifies the noun story. There is no error at (C). The adverb more appropriately modifies the adjective dramatic, which modifies the noun story. There is an error in the sentence. 30 Explanation for Correct Answer C. Choice (C) is correct. The first sentence of the first paragraph states, A castle is not the same thing as a palace. Though the paragraph explains in some detail what a castle is, it does not explain what a palace is. Giving more information about palaces would improve the first paragraph. Choice (A) is unsatisfactory. Although castles were built in the Middle Ages, a brief summary of medieval history would not illustrate the contrast between castles

and palaces, the focus of the topic sentence, and so the summary would not improve the first paragraph. Choice (B) is unsatisfactory. The focus of the first paragraph, as indicated by the topic sentence, is the contrast between castles and palaces. A discussion of the origin of the word castle would not contribute to the topic of the first paragraph and so would not be an improvement. Choice (D) is unsatisfactory. Placing sentence 7 immediately after sentence 1 would create the following statement: A castle is not the same thing as a palace, though some people use the terms castle and palace interchangeably. These had dark dungeons and damp, drafty living quarters instead. In this statement, it is not clear which structure (castles or palaces) had dungeons and unpleasant living quarters. Creating this confusing statement would not improve the first paragraph. Choice (E) is unsatisfactory. The focus of the first paragraph is to show how castles and palaces are different. In sentence 2, castles are referred to as fortified dwellings as a way to highlight this difference. Sentence 3 explains in detail how castles were fortified and the characteristics a castle has that a palace does not. Deleting sentence 3 would make the differences between castles and palaces less apparent and would not improve the first paragraph. 31 Explanation for Correct Answer C. The correct choice is (C). Sentence 3 describes features of a castle such as moats, drawbridges, and walls, which were designed to keep attackers far away from the castle grounds. Sentence 4 describes the defensive methods soldiers used if attackers were able to get closer to the castle. A sentence such as Those who defied such obstacles did so at their peril links the ideas of sentence 3 with those of sentence 4. Choice (A) is unsatisfactory. While sentence 3 mentions the stone walls of a castle, the walls are only one of a number of features described in the sentence. In addition, there is no clear reason to introduce the idea of serfs when sentence 4 continues the idea of castle defenses. A sentence focusing specifically on the walls and the serfs who built them would not be an effective link between sentences 3 and 4. Choice (B) is unsatisfactory. While sentence 3 mentions drawbridges, they are only one of a number of castle features described in the sentence. Sentence 4 elaborates on the idea of castle defenses by describing the maneuvers taken from inside the castle, so a sentence that just describes drawbridges in more detail would not be an effective link between sentences 3 and 4. Choice (D) is unsatisfactory. Sentence 3 describes the defensive features of a castles exterior, and sentence 4 describes the methods of defense taken from within the castle. A sentence about land ownership would not effectively link sentences 3 and 4.

Choice (E) is unsatisfactory. Sentence 3 describes the defensive features of a castles exterior, and sentence 4 describes the methods of defense taken from within the castle against hostile visitors. A sentence mentioning visitors without hostile intentions would not be an effective link between sentences 3 and 4. 32 Explanation for Correct Answer E. The correct choice is (E). This sentence continues the discussion of the uninviting features of castles, leading to a logical comparison of castles, with their dark dungeons and damp, drafty living quarters, with the more comfortable royal palaces. Choice (A) is unsatisfactory. This way of combining sentences 6 and 7 implies that royal palaces offered more comforts as a result of medieval castles having dark dungeons and damp, drafty living quarters. Sentences 6 and 7 should be combined in a way that shows palaces were more comfortable than castles because of the different features of each, not one as a result of the other. Choice (B) is unsatisfactory. This way of combining sentences 6 and 7 awkwardly introduces the first discussion of the features of palaces as a modifying clause, interrupting the natural flow of the paragraph. Also, the use of instead implies that medieval castles offered dark dungeons and damp, drafty living quarters as an alternative to the comforts of palaces, an illogical statement. Choice (C) is unsatisfactory. This method of combining sentences 6 and 7, with the use of only, implies that dark dungeons and damp, drafty living quarters were the only features offered by castles to welcomed guests, an illogical idea not present in the original sentences. Choice (D) is unsatisfactory. Combining sentences 6 and 7 in this way is awkward and illogical. For one thing, the new sentence refers to dark dungeons and damp, drafty living quarters without directly stating these are features of castles. For another, this sentence compares palaces as a whole with the dungeons and living quarters inside castles instead of comparing palaces with castles. 33 Explanation for Correct Answer C. The correct choice is (C). The intended result of building castles was to keep people away; the actual result is that the castles now attract many tourists. This opposition between the intended result and the actual result can be described as ironic, so Ironically is an appropriate way to begin sentence 9. Choice (A) is unsatisfactory. Beginning a sentence with the adverb Actually implies that the sentence preceding is a misunderstanding or a mistaken idea.

Sentence 9 is not a restatement or a correction of sentence 8; therefore, inserting Actually at the beginning of sentence 9 would not be appropriate. Choice (B) is unsatisfactory. Beginning a sentence with the adverb Basically implies that the sentence is a more basic or concise phrasing of a previously stated idea. Sentence 8 is about the fact that many castles have survived to the present day. Describing the relationship between the intended and actual results of building castles (as in sentence 9) is not a more concise phrasing of the idea in sentence 8, so inserting Basically at the beginning of sentence 9 would not be appropriate. Choice (D) is unsatisfactory. Beginning sentence 9 with the phrase By contrast implies that the sentence contrasts with or is in opposition to the preceding sentence. Both sentences 8 and 9 focus on medieval castles and their present-day significance there is no contrast, so inserting By contrast at the beginning of sentence 9 would not be appropriate. Choice (E) is unsatisfactory. Beginning a sentence with the phrase In retrospect implies that the sentence makes a statement about the past based on present-day knowledge. Sentence 9 contrasts the intentions of the past with the results of the present, but there is no new statement being made about the past based on presentday knowledge. Hence inserting In retrospect at the beginning of sentence 9 would not be appropriate. 34 Explanation for Correct Answer D. The correct choice is (D). The sentence Others crumble away in relative obscurity provides a transition between sentence 11, which describes two well-maintained castles, and sentence 12, which describes a castle that is reduced to decaying remnants. Choice (A) is unsatisfactory. Inserting the sentence But not all castles can be considered obsolete after sentence 11 implies that the castles described in sentence 11 are considered obsolete. But the castles mentioned in sentence 11 are described as well-maintained tourist attractions, so it would be inappropriate to describe them as obsolete or no longer in use. Choice (B) is unsatisfactory. Inserting the sentence Elsewhere, the environment may be a factor between sentences 11 and 12 implies that the castle in sentence 12 has decayed because of the environment. But nothing in sentence 12 indicates that the environment played a key role in the decay of the castle in the Welsh village. Choice (C) is unsatisfactory. While the sentence However, many tourists prefer to avoid such tourist traps continues the idea of castles as tourist sites from sentence 11, inserting this sentence between sentences 11 and 12 does not provide the necessary transition between the idea of castles as well-maintained tourist

attractions in sentence 11 and the idea of the decaying remnants of a castle in sentence 12. Choice (E) is unsatisfactory. Inserting the sentence Besides, appearances are often misleading between sentences 11 and 12 does not provide an effective transition between the idea of castles as well-maintained tourist attractions in sentence 11 and the idea of the decaying remnants of a castle in sentence 12. The contrast introduced in sentence 12 is the contrasting condition of a different castle, not the difference between the appearance and actual state of a castle. 35 Explanation for Correct Answer B. The correct choice is (B). Sentence 12 describes the contrast between the decaying remnants of a castle and cozy brick houses that sit on the same street. The phrase medieval austerity stands in bold relief against a background of modern comfort appropriately explains the contrast between the decaying remnants and the cozy houses. Choice (A) is unsatisfactory. The second paragraph focuses on castles and their present-day significance. The sentence In the final analysis, palaces are actually little more than luxurious castles addresses the topic of palaces versus castles instead and would not effectively develop the idea of sentence 12 or any other sentence in the paragraph. Choice (C) is unsatisfactory. The second paragraph focuses on castles and their present-day significance. The sentence The decline of the castles importance as an architectural form coincided with the transformation of medieval society addresses the new topic of architecture and medieval society and would not effectively develop the idea of sentence 12 or any other sentence in the paragraph. Choice (D) is unsatisfactory. The second paragraph focuses on medieval castles and their present-day significance. The sentence In the United States, imposing structures like Hearsts Castle are not really castles in the strictest sense of the word addresses the new topic of castle-like structures and would not effectively develop the idea of sentence 12 or any other sentence in the paragraph. Choice (E) is unsatisfactory. While Eilean Donan Castle, a tourist attraction, is mentioned in sentence 11, sentence 12 moves the topic forward to describe the contrast between the decaying remnants of a castle and cozy brick houses existing on the same street. Referring to how Eilean Donan Castle was named does not continue to move the topic forward and would not be an effective way to follow sentence 12.

Section #9: View Explanations

1 Explanation for Correct Answer B. Choice (B) is correct. Straightforward means clear-cut and precise. The sentence describes Richard Wrights prose as being free of stylistic tricks or evasiveness. In other words, the book is written in a way that is clear and precise; the book is straightforward. Choice (A) is incorrect. Imprecise means vague or not exact. The sentence describes Richard Wrights prose as being free of stylistic tricks or evasiveness. In other words, the book is written in a way that is clear and precise. Something that is vague or not exact is the opposite of something that is precise, so the term imprecise does not logically complete the sentence. Choice (C) is incorrect. Deficient means lacking in some necessary quality or element. Although the sentence describes Richard Wrights prose as being free of certain things, the term deficient is not the best choice; nothing suggests that stylistic tricks and evasiveness are necessary elements that Wrights prose lacks. On the contrary, evasiveness is something a writer might try to avoid. Choice (D) is incorrect. Obtrusive means undesirably prominent or noticeable. The sentence describes the prose of Richard Wrights autobiography as free of stylistic tricks or evasiveness. Stylistic tricks might be described as undesirably noticeable, but it is unlikely that one would describe prose that does not contain such tricks as obtrusive. Choice (E) is incorrect. Something that is elliptical is deliberately obscure, or not clearly expressed. The sentence describes Richard Wrights prose as being free of stylistic tricks or evasiveness. In other words, the book is written in a way that is clear and precise. If the prose is precise and not evasive, it does not make sense to describe it as deliberately obscure. Therefore, the term elliptical does not logically complete the sentence. 2 Explanation for Correct Answer C. Choice (C) is correct. Enormous means marked by extraordinarily great size, number, or degree. Deafening means very loud. The sentence suggests that the size of the crowd and the resonance of the cheers indicate a resurgence, or renewal, of the teams popularity. An enormous crowd whose cheers were loud and echoing would certainly indicate that the team was popular. Choice (A) is incorrect. Vast means very great in size. In this context, hollow means lacking real value, sincerity, or substance. The sentence suggests that the size of the crowd and the resonance of the cheers indicate a resurgence, or renewal, of the teams popularity. A popular team would certainly draw a great crowd, so the term vast makes sense. But the term hollow does not logically

complete the sentence; if the team was popular, the crowds cheers would probably not have been hollow, or insincere. Choice (B) is incorrect. Something that is sparse consists of few and scattered elements. Thunderous means great or intense. The sentence suggests that the size of the crowd and the resonance of the cheers indicate a resurgence, or renewal, of the teams popularity. The terms sparse and thunderous do not logically complete the sentence. A popular team would probably draw a large crowd, not a small, scattered crowd, and it is somewhat unlikely that a small crowds cheers would be described as thunderous. Choice (D) is incorrect. Unimpressive means lacking the power to excite attention, awe, or admiration. Muted means toned down or subdued. The sentence suggests that the size of the crowd and the resonance of the cheers indicate a resurgence, or renewal, of the teams popularity. The terms unimpressive and muted do not logically complete the sentence. A popular team would likely draw a crowd of an impressive size that cheers loudly, not a crowd of an unimpressive size whose cheers are muted, or subdued. Choice (E) is incorrect. Negligible means so small or unimportant as to warrant little or no attention. Rousing means lively, or giving rise to excitement. The sentence suggests that the size of the crowd and the resonance of the cheers indicate a resurgence, or renewal, of the teams popularity. The terms negligible and rousing do not logically complete the sentence; even a very small crowd could produce a lively cheer, but it is illogical to suggest that a negligible crowd would indicate that the team was popular. 3 Explanation for Correct Answer B. Choice (B) is correct. Altered means changed. In this context, revolution means a fundamental or radical change. The sentence indicates two cause and effect relationships: evidence regarding the expansion of the universe had a certain effect on the perception, or concept, of the universe; then, this new concept had a certain effect on astronomical thinking. The missing terms must work together to illustrate these cause and effect relationships. Together, the terms altered and revolution logically complete the sentence: the evidence altered our perception of the universe, and this change led to a fundamental change in astronomical thinking. Choice (A) is incorrect. In this context, to advance something is to accelerate its growth or progress. A setback is a sudden stoppage of progress. The sentence indicates two cause and effect relationships: evidence regarding the expansion of the universe had a certain effect on the perception, or concept, of the universe; then, this new concept had a certain effect on astronomical thinking. The missing terms must work together to illustrate these cause and effect relationships. The terms advanced and setback do not logically complete the sentence. If the evidence

accelerated the progress of our perception of the universe, it does not make sense to say that the evidence led to a sudden stop in the progress of astronomical thinking. Choice (C) is incorrect. Contradicted means implied the opposite of something. A truce is an agreement to a suspension of fighting. The sentence indicates two cause and effect relationships: evidence regarding the expansion of the universe had a certain effect on the perception, or concept, of the universe; then, this new concept had a certain effect on astronomical thinking. The missing terms must work together to illustrate these cause and effect relationships. The terms contradicted and truce do not logically complete the sentence. The evidence may have contradicted our concept of the universe, but it is somewhat illogical to suggest that a contradiction would lead to a truce. Additionally, nothing in the sentence suggests that a truce would be neededthere is no indication that there was any sort of fighting going on concerning astronomical thinking. Choice (D) is incorrect. Reinforced means strengthened by additional support. A crisis is an unstable or crucial time in which a decisive change is about to occur. The sentence indicates two cause and effect relationships: evidence regarding the expansion of the universe had a certain effect on the perception, or concept, of the universe; then, this new concept had a certain effect on astronomical thinking. The missing terms must work together to illustrate these cause and effect relationships. The terms reinforced and crisis do not logically complete the sentence. Evidence that provided additional support to strengthen our perception of the universe most likely would not lead to a crisisthere would be no need to make any decisive changes. Choice (E) is incorrect. Halted means brought to a stop. A breakthrough is a sudden advance. The sentence indicates two cause and effect relationships: evidence regarding the expansion of the universe had a certain effect on the perception, or concept, of the universe; then, this new concept had a certain effect on astronomical thinking. The missing terms must work together to illustrate these cause and effect relationships. The terms halted and breakthrough do not logically complete the sentence. If the evidence brought our perception of the universe to a stop, there probably would not be a breakthrough, or sudden advance, in astronomical thinking. 4 Explanation for Correct Answer E. Choice (E) is correct. In this context, currency means acceptance or general use. If something is inconclusive, it leads to no definitive conclusion and is unable to end a debate. The structure of the sentence suggests that the theory has gained something, but has not gained it completely because of the evidence. The term currency fits into the first blank, because it makes sense to say that a theory could gain acceptance, and the term inconclusive fits into the second blank, because evidence can certainly be inconclusive, or not lead to a definitive conclusion. The two terms logically complete the sentence: the theory that lead poisoning led to the

decline of the Roman Empire has gained acceptance, but because the evidence is inconclusive, the theory cannot be proven. Choice (A) is incorrect. Credence means credibility or acceptance. Irrefutable means impossible to refute or disprove. The structure of the sentence suggests that the theory has gained something, but has not gained it completely because of the evidencethere is something problematic about the evidence. The term credence fits into the first blank, because it makes sense to say that a theory could gain acceptance, but the term irrefutable does not fit into the second blank. Irrefutable evidence provides confirmation or proof of any theory: it is not problematic in any way. Choice (B) is incorrect. Disrepute is the lack or decline of a good reputation. Dubious means producing doubt or uncertainty. The use of the word although at the beginning of the sentence suggests that the second term shows the limitations of the first term. That is, the theory may have gained a poor reputation, but the structure of the sentence indicates that there is something preventing it from becoming completely disreputable. As a result, it would be illogical to suggest that dubious evidence makes a theory less disreputable. Rather, evidence that produces doubt would support the theorys disrepute. Choice (C) is incorrect. Something that has acceptance is satisfactory or has approval. Convincing means conveying truth or rightness. The structure of the sentence suggests that the theory has gained something, but has not gained it completely because of the evidencethere is something problematic about the evidence. The term acceptance fits into the first blank, because it makes sense to say that a theory could gain approval, but the term convincing does not fit into the second blank. Convincing evidence provides solid support for a theory and should not be problematic in any way. Choice (D) is incorrect. Something with momentum has strength or force. Systematic means having a coherent and regular structure. The use of the word although at the beginning of the sentence suggests that the second term shows the limitations of the first term. That is, the theory may have strength or force, but the structure of the sentence indicates that there is something about the "evidence" that is holding the theory back or weakening it. It would be illogical to suggest that systematic evidence makes a theory weaker. Rather, coherent evidence would support the theorys strength. 5 Explanation for Correct Answer B. Choice (B) is correct. A diaphanous fabric has a very delicate texture that enables one to see through it. The structure of the sentence indicates that the phrase virtually transparent describes the fabrics that the designer favored. Because virtually transparent fabric would be almost entirely see-through, the term diaphanous is the term that logically completes the sentence.

Choice (A) is incorrect. Palpable means capable of being touched or easy to perceive. The structure of the sentence indicates that the phrase virtually transparent describes the fabrics that the designer favored. Because fabrics that can be felt or easily perceived might also be thick and not see-through, the term palpable does not logically complete the sentence. Choice (C) is incorrect. Variegated means having different markings or colors, so variegated fabrics would be very colorful. The structure of the sentence indicates that the phrase virtually transparent describes the fabrics that the designer favored. Fabrics that are decorated with many colors are not necessarily see-through, so the term variegated does not logically complete the sentence. Choice (D) is incorrect. A luxury is something that provides great satisfaction or comfort, but is not necessary. Luxurious fabrics, then, would be very pleasurable. The structure of the sentence indicates that the phrase virtually transparent describes the fabrics that the designer favored. Fabrics that are luxurious or very pleasurable are not necessarily see-through, so the term luxurious does not logically complete the sentence. Choice (E) is incorrect. Something that is anomalous is characterized by irregularity or deviation from what is considered normal. The sentence does not offer rules for what fabrics are considered normal, so it is not possible to determine what the designer would consider abnormal, or anomalous. Further, the structure of the sentence indicates that the phrase virtually transparent describes the fabrics that the designer favored; there is no logical connection between a fabric that is virtually transparent and a fabric that is anomalous. 6 Explanation for Correct Answer D. Choice (D) is correct. Because the term iconoclast describes a person who rejects tradition, attacking common beliefs and powerful institutions, it certainly makes sense to describe Professor Williams as an iconoclast. Choice (A) is incorrect. An "egalitarian" believes in and works towards the goal of equality for all people. It is possible that Professor Williams was an egalitarian, attacking beliefs and institutions that she considered to be opposed to equality, but there is no way to conclude this from the information provided in the sentence. Choice (B) is incorrect. A "dowager" is a widow or elderly woman. It is possible that Professor Williams was a dowager, but nothing in the sentence suggests this. Choice (C) is incorrect. A "dilettante" is a person who has a superficial knowledge of and interest in many different subjects. Being a dilettante has nothing to do with attacking beliefs and institutions, so there is no reason to believe such behavior would earn Professor Williams a reputation as a dilettante.

Choice (E) is incorrect. A "purveyor" is someone who supplies goods or provisions to others, usually by selling them. There is no logical connection between selling goods and attacking beliefs and institutions, so there is no reason to believe that Professor Williams behavior would earn her a reputation as a purveyor. 7 Explanation for Correct Answer C. Choice (C) is correct. The first paragraph of the passage states that Miss Keeldar and her uncle have extremely different "characters"that is, they have very different personalities, temperaments, and values. The rest of the passage shows these differences in action: Mr. Sympson believes that Miss Keeldar should marry a man she does not like just because the man has wealth and connections; Miss Keeldar vows that she will marry only for love. Clearly this is a confrontation between two people whose differences seem irreconcilable, or unable to be resolved. Choice (A) is incorrect. There is no indication that the relationship between Miss Keeldar and her uncle has ever been warm or friendly. The first sentence states that "Miss Keeldar and her uncle had characters that would not harmonize,that never had harmonized." Mr. Sympson's attitude towards his niece is revealed at the end of the second paragraph: he wants to "give her in charge to a proper husband, and wash his hands of her for ever"that is, he wants to have nothing more to do with her. Such feelings are not characteristic of a warm family relationship. Choice (B) is incorrect. This episode would not be best described as a "disappointment" for Miss Keeldar. Her reaction to her uncle's proposal is better described as hostile or contemptuous: her eyes are large, dilated, unsmiling" as she commands her uncle ("Cease to annoy me with the subject: I forbid it!"). Miss Keeldar is angry, not disappointed. Further, a description of the episode should mention both characters. Choice (D) is incorrect. It would be wrong to describe this episode as a collaboration (a collaboration involves working together) between two people with similar goals. Miss Keeldar and her uncle have very different goals and opinions: Miss Keeldar says she will marry only for love, while Mr. Sympson insists she marry a man he deems suitable. Choice (E) is incorrect. Although this conversation does take place within an extended familyMiss Keeldar is Mr. Sympson's nieceit cannot be said to be "about the need for unity." Miss Keeldar and Mr. Sympson do not seem interested in family unity; on the contrary, Mr. Sympson wishes to wash his hands of [Miss Keeldar] for ever, and Miss Keeldar insists that she will not follow her uncles orders. 8 Explanation for Correct Answer C.

Choice (C) is correct. In lines 3-5, Mr. Sympson is described as irritable, despotic, and worldly. The rest of the passage supports this image of Mr. Sympson as an ill-tempered, controlling figure who approaches things practically he cannot understand why Miss Keeldar objects to marrying a man he has deemed suitable and is quite frustrated by her refusal. In lines 3-5, Miss Keeldar is described as having a very different character from her uncle: she is spirited, likes freedom, and is perhaps, romantic. The rest of the passage supports the idea that where Mr. Sympson is rigid and practical, Miss Keeldar is passionate and unconventional: she is resolved to love with [her] whole heart and refuses to marry a man she does not love, even though he has money, connections, and respectability. Many people of the time would indeed see this decision as unconventional, or out of the ordinary. Choice (A) is incorrect. Lines 3-5 describe differences between Miss Keeldar and Mr. Sympson. Mr. Sympson is described as being irritable, despotic, and worldlyin other words, he is ill-tempered, controlling, and approaches things practically. In contrast, Miss Keeldar is described as being spirited, liking freedom, and being perhaps, romanticsuggesting that she is passionate and unconventional where her uncle is rigid and practical. We later see Miss Keeldars passion in her vow to love with [her] whole heart before she marries, and her refusal to marry a man she does not loveeven though he has been deemed suitable. Many people of the time would indeed see this decision as unconventional, or out of the ordinary. Miss Keeldar may in fact seem attractive and mysterious to others (such as Sam Wynne), but the passage does not indicate this. Choice (B) is incorrect. Lines 3-5 describe differences between Miss Keeldar and Mr. Sympson. Mr. Sympson is described as being irritable, despotic, and worldlyin other words, he is ill-tempered, controlling, and approaches things practically. In contrast, Miss Keeldar is described as being spirited, liking freedom, and being perhaps, romanticsuggesting that she is passionate and unconventional where her uncle is rigid and practical. Although Miss Keeldar does express her intent to love a man before she marries him, the passage does not suggest that she is overly concerned with finding a good husband. Choice (D) is incorrect. Lines 3-5 describe differences between Miss Keeldar and Mr. Sympson. Mr. Sympson is described as being irritable, despotic, and worldlyin other words, he is ill-tempered, controlling, and approaches things practically. In contrast, Miss Keeldar is described as being spirited, liking freedom, and being perhaps, romanticsuggesting that she is passionate and unconventional where her uncle is rigid and practical. Miss Keeldar is resolved to love with [her] whole heart, so she might enjoy reading books and poetry about love, but the passage does not indicate this. Choice (E) is incorrect. Lines 3-5 describe differences between Miss Keeldar and Mr. Sympson. Mr. Sympson is described as being irritable, despotic, and worldlyin other words, he is ill-tempered, controlling, and approaches things

practically. In contrast, Miss Keeldar is described as being spirited, liking freedom, and being perhaps, romanticsuggesting that she is passionate and unconventional where her uncle is rigid and practical. Nothing in the passage suggests that Miss Keeldar is featured in any stories, exaggerated or not. 9 Explanation for Correct Answer E. Choice (E) is correct. Mr. Sympson and his niece, Miss Keeldar, have characters that would not harmonizein other words, their personalities are very different and they often do not get along. Because he and Miss Keeldar have such conflicting demeanors and viewpoints, Mr. Sympson is eager to give her in charge to a proper husband, and wash his hands of her for ever. He desperately wants to free himself of any responsibility for Miss Keeldar; he wants her to get married as soon as possible so her husband can be the one to take care of her. Choice (A) is incorrect. Mr. Sympson and his niece, Miss Keeldar, have characters that would not harmonizein other words, their personalities are very different and they often do not get along. The passage states that Mr. Sympson is eager to give her in charge to a proper husband, and wash his hands of her for ever. In other words, he no longer wants any responsibility for Miss Keeldar; in fact, he no longer wants her in his life. It is unlikely, then, that he is anxiously anticipating a project on which he and his niece can collaborate, or work together. He is merely looking for the quickest and easiest way to get rid of her. Choice (B) is incorrect. Mr. Sympson and his niece, Miss Keeldar, have characters that would not harmonizein other words, their personalities are very different and they often do not get along. Rather than wanting to help his headstrong niece to realize her ambitious goals, such as marrying for love rather than for money, Mr. Sympson is eager to give her in charge to a proper husband, and wash his hands of her for ever. In other words, he no longer wants any responsibility for Miss Keeldar; in fact, he no longer wants her in his life. He is merely looking for the quickest and easiest way to get rid of her. Choice (C) is incorrect. The passage does suggest that Mr. Sympson is somewhat concerned about the familys reputation: he wants to make for her a suitable match and he warns Miss Keeldar not to sully the family name by a misalliance. However, the passage also states that Mr. Sympson anxiously desires to give her in charge to a proper husband, and wash his hands of her for ever. In other words, he no longer wants any responsibility for Miss Keeldar; in fact, he no longer wants her in his life. His concern for his familys reputation is overshadowed by his primary motivation to get rid of her completely. Choice (D) is incorrect. Mr. Sympson and his niece, Miss Keeldar, have characters that would not harmonizein other words, their personalities are very different and they often do not get along. Because he and Miss Keeldar have such conflicting demeanors and viewpoints, Mr. Sympson is eager to give her in charge to a proper

husband, and wash his hands of her for ever. In other words, he no longer wants any responsibility for Miss Keeldar; in fact, he no longer wants her in his life. While Miss Keeldar herself may be frustrated with the limited opportunities available, Mr. Sympson is not: he considers marriage to Samuel Fawthrop Wynne a "suitable" opportunity for his niece. 10 Explanation for Correct Answer E. Choice (E) is correct. Mr. Sympson states that Mr. Wynne would be a suitable and proper husband because he has a fine unencumbered estate; real substance; good connections. Clearly, Mr. Sympsons definition of a suitable marriage focuses on the social and financial advantagesa nice home, wealth, relationships with influential peoplethat would present themselves if Miss Keeldar were to marry Mr. Wynne. Choice (A) is incorrect. Mr. Sympson states that Mr. Wynne would be a suitable and proper husband because he has a fine unencumbered estate; real substance; good connections. Clearly, Mr. Sympsons definition of a suitable marriage focuses on the social and financial advantagesa nice home, wealth, relationships with influential peoplethat would present themselves if Miss Keeldar were to marry Mr. Wynne. Nothing in the passage indicates that Mr. Sympson gives any thought to the emotional needs of either partner in a marriage. Choice (B) is incorrect. Mr. Sympson states that Mr. Wynne would be a suitable and proper husband because he has a fine unencumbered estate; real substance; good connections. Clearly, Mr. Sympsons definition of a suitable marriage focuses on the social and financial advantagesa nice home, wealth, relationships with influential peoplethat would present themselves if Miss Keeldar were to marry Samuel Wynne. Nothing in the passage indicates that Mr. Sympson gives any consideration to how a marriagesuitable or otherwisewill benefit the local community. Choice (C) is incorrect. Mr. Sympson states that Mr. Wynne would be a suitable and proper husband because he has a fine unencumbered estate; real substance; good connections. Clearly, Mr. Sympsons definition of a suitable marriage focuses on the social and financial advantagesa nice home, wealth, relationships with influential peoplethat would present themselves if Miss Keeldar were to marry Samuel Wynne. Mr. Sympson does not mention the necessity for all family members to approve of a marriage. Choice (D) is incorrect. Mr. Sympson states that Mr. Wynne would be a suitable and proper husband because he has a fine unencumbered estate; real substance; good connections. Clearly, Mr. Sympsons definition of a suitable marriage focuses on the social and financial advantagesa nice home, wealth, relationships with influential peoplethat would present themselves if Miss Keeldar were to marry Samuel Wynne. Mr. Sympson does not mention formal ceremonies or celebrations.

11 Explanation for Correct Answer A. Choice (A) is correct. Miss Keeldars first objection to Mr. Wynne is that he has run a course of despicable, commonplace profligacy. A person who exhibits profligacy behaves in a wildly extravagant and wasteful manner. In other words, Miss Keeldar thinks Mr. Wynne wastes his time in reckless, undignified pursuits. Choice (B) is incorrect. Miss Keeldars first objection to Mr. Wynne is that he has run a course of despicable, commonplace profligacy. A person who exhibits profligacy behaves in a wildly extravagant and wasteful manner. Mr. Wynnes actions could certainly cause damage, and he may not necessarily express any regret for this damage; however, Miss Keeldar does not comment on any damages caused or on Mr. Wynnes response to the consequences of his actions. She simply states her objection based on the reckless nature of his actions. Choice (C) is incorrect. Miss Keeldars first objection to Mr. Wynne is that he has run a course of despicable, commonplace profligacy. A person who exhibits profligacy behaves in a wildly extravagant and wasteful manner. It is possible that Mr. Wynne, through his reckless actions, fails to treat others with the respect they deserve. However, Miss Keeldar does not comment on Mr. Wynnes interactions with others or on the effects of Mr. Wynnes actions on others. She simply states her objection based on the reckless nature of his actions. Choice (D) is incorrect. Miss Keeldars first objection to Mr. Wynne is that he has run a course of despicable, commonplace profligacy. A person who exhibits profligacy behaves in a wildly extravagant and wasteful manner. Mr. Wynne could certainly be dependent on his family for financial support because of his wasteful tendencies, but Miss Keeldar does not indicate this in her objection. She merely comments on the reckless nature of his actions. Choice (E) is incorrect. Later in the passage, Miss Keeldar does remark that Mr. Wynne is lacking in abilities related to literature and art (He cannot write a note without . . . errors; he reads only a sporting paper). But in lines 41-43, Miss Keeldar presents her first objection to Mr. Wynne: he has run a course of despicable, commonplace profligacy. A person who exhibits profligacy behaves in a wildly extravagant and wasteful manner. Miss Keeldar is simply objecting to the reckless nature of Mr. Wynnes actions. 12 Explanation for Correct Answer B. Choice (B) is correct. In lines 54-55, Mr. Sympson poses the following question to Miss Keeldar: Is it your intention ever to marry, or do you prefer celibacy? The question is straightforward; Mr. Sympson is simply asking Miss Keeldar whether she plans to get married or whether she plans to remain unmarried. Miss Keeldar

responds by saying, I deny your right to claim an answer to that question. In other words, Mr. Sympson has no right to ask the question, so Miss Keeldar feels she has no obligation to address the question by providing an answer. Choice (A) is incorrect. In lines 54-55, Mr. Sympson poses the following question to Miss Keeldar: Is it your intention ever to marry, or do you prefer celibacy? The question is straightforward; Mr. Sympson is simply asking Miss Keeldar whether she plans to get married or whether she plans to remain unmarried. He is not accusing her of being secretly engaged, so Miss Keeldar does not respond by denying the accusation. Rather, she responds by insisting that Mr. Sympson has no right to ask the question and that she has no obligation to answer it. Choice (C) is incorrect. In lines 54-55, Mr. Sympson poses the following question to Miss Keeldar: Is it your intention ever to marry, or do you prefer celibacy? The question is straightforward; Mr. Sympson is simply asking Miss Keeldar whether she plans to get married or whether she plans to remain unmarried. There are no exaggerations implicit in the question, so Miss Keeldar does not respond by correcting any exaggerations. Instead, she responds by insisting that Mr. Sympson has no right to ask the question and that she has no obligation to answer it. Choice (D) is incorrect. In lines 54-55, Mr. Sympson poses the following question to Miss Keeldar: Is it your intention ever to marry, or do you prefer celibacy? The question is straightforward; Mr. Sympson is simply asking Miss Keeldar whether she plans to get married or whether she plans to remain unmarried. Although she asserts her right to refuse to marry Sam Wynne, Miss Keeldar does not make, or contradict, any claims about complete independence. Instead, she responds by insisting that Mr. Sympson has no right to ask the question and that she has no obligation to answer it. Choice (E) is incorrect. In lines 54-55, Mr. Sympson poses the following question to Miss Keeldar: Is it your intention ever to marry, or do you prefer celibacy? The question is straightforward; Mr. Sympson is simply asking Miss Keeldar whether she plans to get married or whether she plans to remain unmarried. In the passage, Miss Keeldar does assert her right to refuse to marry Sam Wynne, but she does not respond to her uncles question by asserting her right to live without marrying. Rather, she responds by insisting that Mr. Sympson has no right to ask the question and that she has no obligation to answer it. 13 Explanation for Correct Answer D. Choice (D) is correct. When Mr. Sympson says to Miss Keeldar, be on your guard! I will not be trifled with! he is warning her not to mock him or treat him as unimportant. Throughout the passage, however, it is clear that Mr. Sympson has no real power over Miss Keeldar. She blatantly refuses to marry Sam Wynne; she asserts her right not to answer his inappropriate questions; and she does not let him persuade her to act against her beliefs. Lines 75-76 even suggest that Mr. Sympson

is intimidated by Miss Keeldar, as he unsteadily tries to instill fear in her: Take care! take care! (warning her with voice and hand that trembled alike.). The passage suggests that Mr. Sympsons implied threats are baselessthey are unwarranted and unjustified. Choice (A) is incorrect. When Mr. Sympson says to Miss Keeldar, be on your guard!I will not be trifled with! he is warning her not to mock him or treat him as unimportant. The threat implied by his warning is somewhat implausible, or difficult to believe, but not because Miss Keeldar is able to outwit Mr. Sympson. Rather, the passage suggests that the threat is implausible because Mr. Sympson does not have any real power over Miss Keeldar. She blatantly refuses to marry Sam Wynne; she asserts her right not to answer his inappropriate questions; and she does not let him persuade her to act against her beliefs. She even calls attention to Mr. Sympsons lack of authority: What shadow of power have you over me? Choice (B) is incorrect. When Mr. Sympson says to Miss Keeldar, be on your guard!I will not be trifled with! he is warning her not to mock him or treat him as unimportant. Although Mr. Sympson might very well be serious when he warns Miss Keeldar to be on her guard, the threat implied in his warning is not a serious one; Mr. Sympson has no real power over Miss Keeldar's decisions. In fact, Mr. Sympson wishes that he had more authority over Miss Keeldar so that he could just force her to marry Sam Wynne: Why are not the laws more stringent, that I might compel her to hear reason? Choice (C) is incorrect. When Mr. Sympson says to Miss Keeldar, be on your guard!I will not be trifled with! he is warning her not to mock him or treat him as unimportant. The threat implied by his warning is certainly not misleadinghe is truly attempting to threaten her in some way. Further, the passage indicates that Mr. Sympson is not especially concerned with Miss Keeldars happiness. He is simply trying to give her in charge to a proper husband, and wash his hands of her for ever. He doesnt seem to care whether or not she has a happy marriage; he just doesnt want to have responsibility for her anymore. Choice (E) is incorrect. When Mr. Sympson says to Miss Keeldar, be on your guard!I will not be trifled with! he is warning her not to mock him or treat him as unimportant. Indeed, the threat implied by his warning is somewhat absurd; he does not have any real power over her. Mr. Sympson may be trying to intimidate Miss Keeldar, but she is not making an effort to intimidate him. Throughout the passage Miss Keeldar asserts herself, voicing her objections to Sam Wynnes character and expressing her desire to love a man before she marries him, but she does not attempt to exert any power over Mr. Sympson. She simply insists that she will act as she sees fit. 14 Explanation for Correct Answer A.

Choice (A) is correct. Trying to defend himself and prove to Miss Keeldar that he is someone to be feared, Mr. Sympson says things like, be on your guard!I will not be trifled with! and Take care, madam! In response, Miss Keeldar says, Scrupulous care I will take, Mr. Sympson. Before I marry, I am resolved to esteem to admireto love. Instead of taking care, or being careful, not to belittle or anger him, she deliberately misunderstands his meaning. Miss Keeldar claims that she will indeed take care; that is, she will make sure she loves a man before she marries him, rather than carry out Mr. Sympsons wishes and marry Sam Wynne, a man she does not love. Choice (B) is incorrect. Throughout the passage, Miss Keeldar voices her objections to marrying Sam Wynne, ignores Mr. Sympsons attempts to convince her that Wynne is a suitable husband, and points out that Mr. Sympson has no real power over her. There is no specific blame placed, or turned back on, anyone during their discussion. In these lines, Miss Keeldar simply shows her refusal to be intimidated and her desire to love a man before she marries him. Choice (C) is incorrect. Trying to defend himself and prove to Miss Keeldar that he is someone to be feared, Mr. Sympson says things like, be on your guard!I will not be trifled with! and Take care, madam! In response, Miss Keeldar says, Scrupulous care I will take, Mr. Sympson. Before I marry, I am resolved to esteem to admireto love. Although it may appear that Miss Keeldar is mocking the tone of his comment by twisting his words a bit, she is certainly not acting childishly. On the contrary, she is standing by her convictionsshe vows to love a man before she marries him, rather than marry someone she does not even respect just because her uncle deems him a suitable husband. Choice (D) is incorrect. Trying to defend himself and prove to Miss Keeldar that he is someone to be feared, Mr. Sympson says things like, be on your guard!I will not be trifled with! and Take care, madam! In response, Miss Keeldar says, Scrupulous care I will take, Mr. Sympson. Before I marry, I am resolved to esteem to admireto love. Miss Keeldar does not lament Mr. Sympsons failure to sympathize with her; she seems unconcerned with Mr. Sympsons lack of sympathy. In fact, in lines 84-86 Miss Keeldar states that although she may speak in an unknown tongue, she feels indifferent about being understoodin other words, she knows that her plan to marry for love is unusual, but she does not care whether or not others understand her decision. Choice (E) is incorrect. Trying to defend himself and prove to Miss Keeldar that he is someone to be feared, Mr. Sympson says things like, be on your guard!I will not be trifled with! and Take care, madam! In response, Miss Keeldar says, Scrupulous care I will take, Mr. Sympson. Before I marry, I am resolved to esteem to admireto love. She does not refer to any of her previous behavior, sensible or otherwise. Instead she is talking about how she plans to behave in the future, expressing her desire to love a man before she marries him. 15

Explanation for Correct Answer D. Choice (D) is correct. As Miss Keeldar explains to Mr. Sympson that she is resolved to esteemto admireto love the man she intends to marry, she makes the following comment: I know I speak in an unknown tongue; but I feel indifferent whether I am comprehended or not. In this context, the term tongue means language. Miss Keeldar is exaggerating, of course; Mr. Sympson understands the words she is speaking. However, by saying that she is speaking in a language that is unknown to Mr. Sympson, she is making the point that he is incapable of understanding her desire to marry for love. Choice (A) is incorrect. As Miss Keeldar explains to Mr. Sympson that she is resolved to esteemto admireto love the man she intends to marry, she makes the following comment: I know I speak in an unknown tongue; but I feel indifferent whether I am comprehended or not. In this context, the term tongue means language. Miss Keeldar is exaggerating, of course; Mr. Sympson understands the words she is speaking. However, by saying that she is speaking in a language that is unknown to Mr. Sympson, she is making the point that he is incapable of understanding her desire to marry for love. Miss Keeldar's focus is on Mr. Sympson's inability to understand her feelings, not on his mistrust of anything new and unfamiliar. Choice (B) is incorrect. As Miss Keeldar explains to Mr. Sympson that she is resolved to esteemto admireto love the man she intends to marry, she makes the following comment: I know I speak in an unknown tongue; but I feel indifferent whether I am comprehended or not. Earlier in their conversation Miss Keeldar suggests that Mr. Sympson is ignorant of some of Sam Wynnes undesirable qualities, but Wynne is not specifically mentioned in this part of the conversation. Miss Keeldar is not suggesting that her uncle is ignorant of Wynnes reputation in the community; instead, she is suggesting that Mr. Sympson is incapable of understanding her desire to love a man before she marries him. Choice (C) is incorrect. As Miss Keeldar explains to Mr. Sympson that she is resolved to esteemto admireto love the man she intends to marry, she makes the following comment: I know I speak in an unknown tongue; but I feel indifferent whether I am comprehended or not. In this context, the term tongue means language. Rather than suggesting that Mr. Sympson has a tendency to speak in an obscure, or not easily understood, way, Miss Keeldar is suggesting that she is speaking of something Mr. Sympson will not understandher desire to love a man before she marries him. Choice (E) is incorrect. As Miss Keeldar explains to Mr. Sympson that she is resolved to esteemto admireto love the man she intends to marry, she makes the following comment: I know I speak in an unknown tongue; but I feel indifferent whether I am comprehended or not. In this context, the term tongue means language. Miss Keeldar is exaggerating, of course; Mr. Sympson understands the words she is speaking. However, by saying that she is speaking in a language

that is unknown to Mr. Sympson, she is making the point that he is incapable of understanding her desire to marry for love. Nowhere in the passage is there any discussion of other family members or the mistakes they have made. 16 Explanation for Correct Answer B. Choice (B) is correct. Throughout the passage, Mr. Sympson has been trying to convince Miss Keeldar to marry Sam Wynne. She refuses to do so, voicing her many objections regarding Wynnes character and reputation. After expressing his disapproval with Miss Keeldars use of what he considers Unladylike language, Mr. Sympson, frustrated, gestures toward the heavens and asks, To what will she come?implying that, due to her behavior, he thinks Miss Keeldar will come to no good. He poses the question as a prediction of a bleak future for her. Choice (A) is incorrect. Throughout the passage, Mr. Sympson has been trying to convince Miss Keeldar to marry Sam Wynne. She refuses to do so, voicing her many objections regarding Wynnes character and reputation. Mr. Sympsons question is not a warning about financial losses; he does refer to Wynne as a wealthy man, but Miss Keeldar does not stand to lose any money by refusing to marry Wynne. Rather, Mr. Sympson implies that, due to her behavior, Miss Keeldar has a bleak future. Choice (C) is incorrect. Throughout the passage, Mr. Sympson has been trying to convince Miss Keeldar to marry Sam Wynne. She refuses to do so, voicing her many objections regarding Wynnes character and reputation. Mr. Sympson wants Miss Keeldar to marry this suitable match and proper husband, so he is not relieved that Miss Keeldar objects to the marriagefar from it. Rather than confessing his relief, Mr. Sympson asks his question to imply that Miss Keeldar may be facing a bleak future due to her behavior. Choice (D) is incorrect. Throughout the passage, Mr. Sympson has been trying to convince Miss Keeldar to marry Sam Wynne. She refuses to do so, voicing her many objections regarding Wynnes character and reputation. Although Mr. Sympson is clearly frustrated with Miss Keeldar and the conversation, he does not plead for a diversion; on the contrary, he likely wants to continue the conversation so that he might convince Miss Keeldar to marry Wynne. Mr. Sympson asks his question to imply that Miss Keeldar may be facing a bleak future due to her behavior. Choice (E) is incorrect. Throughout the passage, Mr. Sympson has been trying to convince Miss Keeldar to marry Sam Wynne. She refuses to do so, voicing her many objections regarding Wynnes character and reputation. Rather than condemning conventional lifestyles, Mr. Sympson actually wants Miss Keeldar to have a conventional lifestyleto marry Wynne because he is a suitable match and a proper husband. Mr. Sympsons question in line 100 implies that he thinks Miss

Keeldar will come to no good. He poses the question as a prediction of a bleak future for her. 17 Explanation for Correct Answer C. Choice (C) is correct. In response to Mr. Sympsons question, To what will she come? Miss Keeldar says, Never to the altar with Sam Wynne. Throughout the passage, Mr. Sympson is attempting to convince Miss Keeldar that she should marry Sam Wynne. When Mr. Sympson gets frustrated and asks his rhetorical question that is, a question to which he does not expect an answerMiss Keeldar responds as if he were sincerely inquiring about her future plans. She says that her plans will never include marrying Sam Wynne. Choice (A) is incorrect. In response to Mr. Sympsons question, To what will she come? Miss Keeldar says, Never to the altar with Sam Wynne. Earlier in the passage, in line 56, Miss Keeldar asserted her right to ignore a question posed by Mr. Sympsona question she felt was inappropriate. But in line 100, Mr. Sympsons question is rhetoricalthat is, it is a question to which he does not expect an answer. Miss Keeldar responds as if her uncle were sincerely inquiring about her future plans, saying that her plans will never include marrying Sam Wynne. She does not suggest that the question is an unreasonable request. Choice (B) is incorrect. In response to Mr. Sympsons question, To what will she come? Miss Keeldar says, Never to the altar with Sam Wynne. Mr. Sympsons question is rhetoricalthat is, it is a question to which he does not expect an answer. Miss Keeldar responds to the question as if her uncle were sincerely inquiring about her future plans. She says that her plans will never include marrying Sam Wynne. Her response does not indicate that she sees the question as an appeal to her sense of fair play. Choice (D) is incorrect. In response to Mr. Sympsons question, To what will she come? Miss Keeldar says, Never to the altar with Sam Wynne. Mr. Sympsons question is rhetoricalthat is, it is a question to which he does not expect an answer. While Mr. Sympson may be questioning Miss Keeldars morals, she does not respond to his question as if it were an expression of moral uncertainty. Rather, she responds to the question as if her uncle were sincerely inquiring about her future plans, saying that her plans will never include marrying Sam Wynne. Choice (E) is incorrect. In response to Mr. Sympsons question, To what will she come? Miss Keeldar says, Never to the altar with Sam Wynne. Mr. Sympsons question is rhetoricalthat is, it is a question to which he does not expect an answer. Miss Keeldars response does not mention her family at all, so she does not interpret Mr. Sympsons question as an attempt to understand her familys history. Rather, she responds to the question as if her uncle were sincerely inquiring about her future plans, saying that her plans will never include marrying Sam Wynne.

18 Explanation for Correct Answer B. Choice (B) is correct. As Mr. Sympson attempts to convince Miss Keeldar that she should marry Sam Wynne, he argues that Wynne is worthy of Miss Keeldar because he has money, common sense, connections, and respectability. Miss Keeldar does not believe, however, that these aspects of Wynns character make him a suitable husband. She has no desire to marry a man simply because of his financial status or his connections. In lines 80-84, Miss Keeldar states that, before she marries, she is resolved to esteemto admire and to love with [her] whole heart. She believes that love is a crucial prerequisitesomething that is absolutely necessaryfor a satisfactory marriage. Choice (A) is incorrect. In lines 80-84, Miss Keeldar states that, before she marries, she is resolved to esteemto admire and to love with [her] whole heart. She specifically points out that she intends to love the man she marries before getting married, not that love is a natural consequence, or result, of a prolonged companionship. If Miss Keeldar did believe that love could eventually develop from a prolonged companionship, she most likely would be more open to the idea of marrying Sam Wynne, a man she currently does not love. Choice (C) is incorrect. Although Miss Keeldar may believe that love is a desirable element in an independent womans life, she does not necessarily express this belief in the passage. Miss Keeldar does specifically state in lines 80-84 that, before she marries, she is resolved to esteemto admire and to love with [her] whole heart. Miss Keeldars remarks in the passage reveal that she thinks love is absolutely necessary for a satisfactory marriage. Choice (D) is incorrect. In lines 80-84, Miss Keeldar states that, before she marries, she is resolved to esteemto admire and to love with [her] whole heart. She specifically points out that love is something that is necessary before getting married, not necessarily a fortunate accident that sometimes results from marriage. If Miss Keeldar believed that love could result from a marriage, even when it doesnt exist before it, she most likely would be more open to the idea of marrying Sam Wynne, a man she currently does not love. Choice (E) is incorrect. In lines 80-84, Miss Keeldar states that, before she marries, she is resolved to esteemto admire and to love with [her] whole heart. If Miss Keeldar believed that love is a sentimental delusion that is potentially harmful, she probably would not be so committed to making sure that love is a part of her life in particular, that she loves a man before she marries him. She would most likely want to avoid love at all costs. 19 Explanation for Correct Answer E.

Choice (E) is correct. More than once, Mr. Sympson points out Sam Wynnes wealth as proof that he would make a suitable husband, so he clearly thinks Miss Keeldar should not marry someone who is poor. Miss Keeldar agrees, to a point: when asked if her love would ever fall on a beggar, she responds, On a beggar it will never fall. Mendicancy is not estimable. When Mr. Sympson mentions other poor men, such as "a play-actor, a play-writer. . . . [a]ny literary scrub, or shabby, whining artist," Miss Keeldar replies that, despite her love of the arts, she has "no taste" for "the scrubby, shabby, whining." It seems that Miss Keeldar seeks a man of refinement, dignity, and some wealthnot a starving artist. Choice (A) is incorrect. More than once, Mr. Sympson points out Sam Wynnes wealth and his connections as proof that he would make a suitable husband, so he clearly thinks Miss Keeldar should seek marriage with an aristocratic man. Miss Keeldar obviously disagreesshe objects to Sam Wynnes disrespectful behavior and lack of intelligence, and refuses to marry him, even if he is an aristocratic man. Choice (B) is incorrect. Mr. Sympson certainly believes that Miss Keeldar should feel honored by the attentions of the Wynne family. He considers the union of Miss Keeldar and Sam Wynne to be decidedly suitable and most proper. In response to her refusal to consent to the marriage, Mr. Sympson states, In all respects he is more than worthy of you. Miss Keeldar does not feel that she should be honored by such attention. She immediately points out a number of Sam Wynnes negative qualities and characteristics and states that she has absolutely no desire to marry him, despite the fact that he comes from a respected family. Choice (C) is incorrect. Whether or not Miss Keeldar feels she may need to become more mature before she marries, Mr. Sympson does not care whether or not she is mature enough to get married. He is desperately trying to give her in charge to a proper husband, and wash his hands of her for ever. He doesnt want to be bothered with her anymore. Choice (D) is incorrect. It is important to Miss Keeldar that she stands by her beliefs, such as her resolution to esteemto admireto love before she marries, so she refuses to marry Sam Wynne, a man she currently does not love. Mr. Sympson does not care whether or not Miss Keeldar is acting against her most deeply held beliefs. He is desperately trying to give her in charge to a proper husband, and wash his hands of her for ever. He doesnt want to be bothered with her anymore.

Section #10: View Explanations 1 Explanation for Correct Answer D.

Choice (D) is correct. It avoids the error of the original by providing a plural verb (are) that agrees with the plural noun challenges. Choice (A) involves subject-verb disagreement. The singular verb is does not agree with the plural noun challenges. The plural verb are should be used instead. Choice (B) results in subject-verb disagreement and awkward phrasing. The singular verb is does not agree with the plural noun challenges. The plural verb are should be used instead. In addition, the infinitive phrase to associate is awkward in this context and alters the meaning of the original sentence. The past-tense verb associated should be used instead. Choice (C) involves subject-verb disagreement and results in an illogical statement. The singular verb is does not agree with the plural noun challenges. The plural verb are should be used instead. In addition, it does not make logical sense to say that challenges are associating with starting ones own business. While a person or people can associate with other people, a thing, such as challenges, cannot associate with another thing (in this case, the gerund phrase starting ones own business). Choice (E) results in an illogical statement. It does not make logical sense to say that challenges associate with starting ones own business. While a person or people can associate with other people, a thing, such as challenges, cannot associate with another thing (in this case, the gerund phrase starting ones own business). 2 Explanation for Correct Answer C. Choice (C) is correct. It avoids the errors of the original by eliminating the vague pronoun it and by providing an idiomatic and logical preposition to introduce the prepositional phrase that modifies display (on display at the museum). Choice (A) involves a vague pronoun and unidiomatic phrasing. There is no antecedent to which the pronoun it can refer. In addition, it is not idiomatic and logical to say that the watercolors are on display by the museum. The preposition at should be used instead. Choice (B) results in wordiness and subject-verb disagreement. The phrase that it, the museum, is displaying is unnecessarily wordy and should be replaced with that the museum is displaying. In addition, the singular verb represents does not agree with the plural subject watercolors. Choice (D) results in a sentence fragment. There is no main verb to carry out the action of the sentence, only the participle representing.

Choice (E) results in subject-verb disagreement and an unidiomatic conjunction. The singular verb represents does not agree with the plural subject watercolors. In addition, the conjunction while is not idiomatic in this context and should be replaced with when. 3 Explanation for Correct Answer B. Choice (B) is correct. It avoids the error of the original by providing a plural noun (the presidencies) that can be logically modified by the prepositional phrase of Theodore Roosevelt, William Howard Taft, and Woodrow Wilson. Because the sentence is talking about three presidents, the plural presidencies must be used in place of the singular presidency. Choice (A) involves noun-noun disagreement in number. Because the sentence is talking about three presidents, the plural presidencies must be used in place of the singular presidency. Choice (C) results in unidiomatic phrasing and noun-noun disagreement in number. It is not idiomatic to say that the origins of something happened. In this case, the verb phrase can be traced should be used instead. In addition, because the sentence is talking about three presidents, the plural presidencies must be used in place of the singular presidency. Choice (D) involves unidiomatic phrasing and results in a sentence fragment. The participle happening is unidiomatic in this context. In addition, there is no main verb to carry out the action of the sentence, only the participle happening. Choice (E) involves unidiomatic phrasing, results in a sentence fragment, and produces noun-noun disagreement in number. It is not idiomatic to say that the origins of something happened. In this case, the verb phrase can be traced should be used instead. In addition, there is no main verb, only the phrase that happened . . . Further, because the sentence is talking about three presidents, the plural presidencies must be used in place of the singular presidency. 4 Explanation for Correct Answer E. Choice (E) is correct. It avoids the pronoun-reference error of the original by clearly stating who noticed that the menu had changed. Choice (A) involves an unclear pronoun reference. It is not clear to whom the pronoun she is meant to refer. Choice (B) results in unclear pronoun reference and inappropriate shifts in verb tense. It is not clear to whom the pronoun she is meant to refer. In addition, the present-tense verb notices is not parallel with past-tense verb that precedes it

(visited), and the past perfect has changed is not appropriate when describing an action that took place in the past (When Sheila and Lucy visited the restaurant). Choice (C) results in an inappropriate shift in verb tense. While the past perfect has noticed would be appropriate to describe an action that is ongoing, the past-tense noticed is needed instead to indicate an action that took place in the past (When Sheila and Lucy visited the restaurant). Choice (D) results in an illogical statement. It does not make sense to say that while at the restaurant, Sheila had noticed the menu changing. 5 Explanation for Correct Answer C. Choice (C) is correct. It avoids the modification error of the original by providing a noun (dust) that can logically be modified by the sentences introductory phrase, Carried by the strong, dry winds of the stratosphere. Choice (A) involves illogical modification. It does not make sense to say that the 1980 eruption of Mount Saint Helens was Carried by the strong, dry winds of the stratosphere. Choice (B) results in illogical modification. It does not make sense to say that Mount Saint Helens eruption was Carried by the strong, dry winds of the stratosphere. Choice (D) results in a sentence fragment. There is no main verb to carry out the action of the sentence, only the participle Carried and a relative clause beginning with that. Choice (E) results in flawed sentence structure. The introductory phrase Carried by the strong, dry winds of the stratosphere cannot logically modify the word there. 6 Explanation for Correct Answer E. Choice (E) is correct. It avoids the error of the original by providing an appropriate comparative structure (so . . . that) to establish the cause-and-effect relationship between complicating the process of formulating a school budget and the fact that no one on the Board of Education is eager to undertake the task. Choice (A) involves faulty sentence structure. For the sentence to indicate the result of the new regulations being so complicated, it must provide the word that to complete the so . . . that comparative structure. It is unidiomatic to follow the phrase beginning with so complicated . . . with to where.

Choice (B) results in faulty sentence structure. For the sentence to indicate the result of the new regulations being so complicated, it must provide the word that to complete the so . . . that comparative structure. It is unidiomatic to follow the phrase beginning with so complicated . . . with therefore. Choice (C) results in faulty sentence structure. For the sentence to indicate the result of the new regulations being so complicated, it must provide the word that to complete the so . . . that comparative structure. It is unidiomatic to follow the phrase beginning with so complicated . . . with even. Choice (D) results in faulty sentence structure. For the sentence to indicate the result of the new regulations being so complicated, it must provide the word that to complete the so . . . that comparative structure. It is unidiomatic to follow the phrase beginning with so complicated . . . with as. 7 Explanation for Correct Answer D. Choice (D) is correct. It avoids the unidiomatic phrasing of the original by providing the verb phrase is occurring to follow the noun shift. Choice (A) involves awkward phrasing. It is awkward to say The shift . . . is because. In this context, shift is a noun and cannot be the result of other actions. A verb phrase such as is occurring should be used to describe the action of shift. Choice (B) results in awkward phrasing. It is awkward to say The shift . . . is because. In this context, shift is a noun and cannot be the result of other actions. A verb phrase such as is occurring should be used to describe the action of shift. Choice (C) results in unidiomatic phrasing and a lack of parallelism. It is unidiomatic to say is caused from. It would be more idiomatic to replace the preposition from with by. In addition, the verb phrase become more vain is not parallel with the preceding verb phrase (getting fewer cavities). Choice (E) results in an illogical statement and a lack of parallelism. To say The shift from traditional to cosmetic dentistry occurs implies that this shift happens more than once, which is illogical. In addition, the verb phrase become more vain is not parallel with the preceding verb phrase (getting fewer cavities). 8 Explanation for Correct Answer E. Choice (E) is correct. It avoids the error of the original by providing a subject ("many people") that can be logically modified by the sentences opening phrase, "If asked to name a musical group with broad and lasting appeal."

Choice (A) involves illogical modification. It does not make sense to say that "the Beatles," a famous musical group, would be asked to "name a musical group with broad and lasting appeal" and would also be "the choice for many." Choice (B) results in illogical modification. It does not make sense to say that "the Beatles," a famous musical group, would be asked to "name a musical group with broad and lasting appeal" and would also be "chosen by many people." Choice (C) results in illogical modification. It does not make sense to say that "the choice" would be asked to "name a musical group with broad and lasting appeal." Choice (D) results in an inappropriate verb form. The if-clause that starts the sentence ("If asked to name a musical group with broad and lasting appeal") demands a verb in the conditional tense ("would choose"), but the sentence provides a present-tense verb ("choose"). 9 Explanation for Correct Answer E. Choice (E) is correct. It avoids the errors of the original by providing a subject ("the United States and Canada") that can logically be modified by the phrase "to reduce pollution in the Great Lakes" and a main verb ("set") to carry out the action of the sentence. Choice (A) involves illogical modification and results in a sentence fragment. It does not make sense to say that "limits" purposefully did something "to reduce pollution in the Great Lakes." In addition, there is no main verb to carry out the action of the sentence, only the participial phrase "having been." Choice (B) involves illogical modification and results in a sentence fragment. It does not make sense to say that "limits" purposefully did something "to reduce pollution in the Great Lakes." In addition, here "set" functions as an adjective, and so there is no main verb to carry out the action of the sentence. Choice (C) results in illogical modification and an inappropriate verb form. It does not make sense to say that "limits" purposefully did something "to reduce pollution in the Great Lakes." In addition, the past perfect "have been set" would be appropriate to describe an action that is ongoing; however, the action of the sentence takes place in the past ("In 1972"). Choice (D) results in an inappropriate verb form. The past perfect "have set" would be appropriate to describe an action that is ongoing; however, the action of the sentence takes place in the past ("In 1972"). 10 Explanation for Correct Answer A.

Choice (A) is correct. It avoids the errors of the options by providing an idiomatic modifying phrase that is parallel with the main verb phrase supported himself. Choice (B) results in a lack of parallelism and involves unidiomatic phrasing. The phrase already being a certified lawyer is not parallel with the main past-tense verb phrase supported himself. In addition, it is not idiomatic to say already being a certified lawyer. The verb being is unnecessary in this context and should be deleted. Choice (C) results in illogical phrasing. It does not make sense to say that Ruben Blades was certified for being a lawyer. One must be certified in order to be a lawyer; one does not become certified because they are already a lawyer. Choice (D) results in faulty sentence structure. At the very least, the sentence would need the coordinating conjunction and to join the independent clause Ruben Blades was . . . Panama with another independent clause ([he] supported himself by singing . . .). Choice (E) involves an inappropriate verb tense and results in faulty sentence structure. The word already indicates an action that took place in the past, so the past-tense verb was is needed in place of the present-tense verb is. In addition, the sentence would, at the very least, need the coordinating conjunction and to join the independent clause Ruben Blades was . . . Panama with another independent clause ([he] supported himself by singing . . .). 11 Explanation for Correct Answer D. Choice (D) is correct. It avoids the error of the original by replacing the awkward and somewhat vague being that with the more precise because to indicate the cause-effect relationship between the ideas in the sentence (The newspaper business in the United States faces a challenge because it must . . .). Choice (A) involves awkward and imprecise phrasing. The awkward and somewhat vague being that should be replaced by because to indicate the cause-effect relationship between the ideas in the sentence (The newspaper business in the United States faces a challenge because it must . . .). Choice (B) results in awkward and imprecise phrasing. The awkward of it reconciling should be replaced by the precise because it must reconcile to indicate the cause-effect relationship between the ideas in the sentence. In addition, because the phrasing in this option defines rather than describes the challenge the newspaper business faces, the phrase faces the challenge of . . . is more precise than faces a challenge of . . . Choice (C) results in an illogical statement. The word such in the phrase as such is meant to stand in for a noun that precedes it; in this case, The newspaper

business is the only seemingly plausible choice as the referent of such. However, it does not make sense to say that as the newspaper business, it [the newspaper business] must reconcile . . . Choice (E) results in awkward, confusing phrasing. It is not clear what is meant by it has the high-minded goal . . . with the commercial one or what the phrase being reconciled is meant to modify. 12 Explanation for Correct Answer D. Choice (D) is correct. It avoids the error of the original by providing three parallel clauses to indicate why All species of sea turtles are endangered. Choice (A) involves a lack of parallelism. The verb phrase their eggs being disturbed and the noun phrase destruction of nesting habitats are not parallel with the gerund phrase that precedes them (overharvesting of adults). Choice (B) results in a lack of parallelism. In the first two phrases (the adults being overharvested and their eggs disturbed), the sea turtles are the subject. In the third phrase (destroying nesting habitats), the sea turtles cannot logically be the subject. In addition, the third phrase begins with a present-tense verb while the first and second phrases begin with a noun and pronoun, respectively. Choice (C) results in faulty sentence structure. The phrase because the overharvesting of adults is not proper English syntax. The preposition of should be inserted after because to make the phrase structurally complete. Choice (E) results in faulty sentence structure and a lack of parallelism. The phrase because being overharvested is not proper English syntax. The preposition of should be inserted after because to make the phrase structurally complete. In addition, the phrases being overharvested as adults, their eggs being disturbed, and destruction of nesting habits are not parallel. 13 Explanation for Correct Answer C. Choice (C) is correct. It avoids the subject-verb disagreement of the original by providing a plural verb (contribute) that agrees with the plural compound subject (improper dietand lack of exercise). Choice (A) involves subject-verb disagreement. The singular verb contributes does not agree with the plural compound subject (improper diet and lack of exercise).

Choice (B) results in flawed sentence structure. The plural pronoun they is unnecessary here and intrudes between the plural compound subject (improper diet and lack of exercise) and its main verb (contribute). Choice (D) results in a sentence fragment. There is no main verb to carry out the action of the sentence, only the participle contributing. Choice (E) results in subject-verb disagreement. The singular verb has does not agree with the plural compound subject (improper diet and lack of exercise). 14 Explanation for Correct Answer E. Choice (E) is correct. It avoids the error of the original by providing a phrase (delightful because of his imagination) that is parallel with the phrase that precedes it (annoying because of his unpredictability) and coordinates with the adverbial phrase at once (at once annoying . . . and delightful). Choice (A) involves a lack of parallelism and improper coordination. The phrase but his imagination is still a delight is not parallel with the phrase that precedes it (annoying because of his unpredictability) and does not coordinate with the adverbial phrase at once (at once annoying . . . but his imagination). Choice (B) results in a lack of parallelism and improper coordination. The phrase although he is delightfully imaginative is not parallel with the phrase that precedes it (annoying because of his unpredictability) and does not coordinate with the adverbial phrase at once (at once annoying . . . although he is). Choice (C) results in a lack of parallelism and improper coordination. The phrase and he is delightful in his imagination too is not parallel with the phrase that precedes it (annoying because of his unpredictability) and does not coordinate with the adverbial phrase at once (at once annoying . . . and he is). Choice (D) results in awkward phrasing, a lack of parallelism, and improper coordination. The awkward phrase while being imaginative and they are delighted is not parallel with the phrase that precedes it (annoying because of his unpredictability) and does not coordinate with the adverbial phrase at once (at once annoying . . . while being imaginative).

S-ar putea să vă placă și